Download as pdf or txt
Download as pdf or txt
You are on page 1of 128

1

1. Advanced breast Cancer with fixed unilateral breast mass involving the skin. What
is the next step in management?
A. Core biopsy
Answer is: A

2. What is the cause Nipple inversion?


A. Invasive ductal carcinoma
B. Fibroadenoma
C. duct ectasia infections such as acute mastitis, duct ectasia, tuberculosis
D. Mastitis
Answer is: C

3. Color of discharge from duct papilloma?


A. Red
B. Green Creamy mastitis if fever / duct ectasia no fever
Answer is: A

4. Young age came with Breast mass 15x15 cystic and skin normal but stretched what
is the proper management?
A. Simple mastectomy
B. Radical mastectomy
C. Chemotherapy
D. Cystectomy
Answer is: A

5. Patient presented with 4 cm tumor in her breast, turns out it was phyloids, what is
your treatment?
A. Wide tumor excision (WLE) lumpectomy
B. Mastectomy
C. Follow up
Answer is: A

6. Young woman has painful breast lump with redness and tenderness. Vitals are
given and show T 38.9. What will u do?
A. Incision and drainage
B. Excision
C. FNA
Answer is: A (dx abscess)

7. BIRADS stage 5, MGR?


Answer is: Core biopsy, N.B: BIRADS for Breast: 1,2 > regular screening. 3 > short
follow-up 6 months. 4,5 >>> Core Biopsy. 6 >>> Surgery
1 - negative
2 - negative but there is something benign (e.g. calcifications) anyone else looks at it need
to not misunderstood the findings
3 - follow up is needed ( 6 months)
4 - biopsy is needed
5 - malignancy is a must
6 - mammogram for treatment outcome

2
8. A young female patient with clear bilateral breast discharge, she is otherwise
normal... What will you do?
A. Mammography.
B. ultrasound.
C. prolactin level
D. MRI
Answer is: B, if milk discharge > C.

9. A 45-year-old woman complains to her primary care physician of nervousness,


sweating, tremulousness, and weight loss. The thyroid scan reveals a single focus
of increased isotope uptake. Which of the following is the most likely disorder?
A. Hypersecreting adenoma
B. Graves disease
C. Multinodular goiter
D. Papillary carcinoma of thyroid
Answer is: A, N.B: single focus of increased isotope uptake, often referred to
as a hot nodule.

10. Patient was doing US to neck and accidentally discover nodule it Rt lobe, low TSH
HIGH T4 what is the proper investigation?
A. Radioactive iodine scan
B. Hemithyroidectomy
C. FNA
D. Follow up
Answer is: A

11. Thyroid nodule FNA was done with result follicular carcinoma grade 4 Radioactive
iodine what is the proper next step?
A. Hemithyroidectomy
B. Repeat FNA after 3 months
C. Total thyroidectomy
Answer is: A

12. Bethesda Stage 4 management?


Answer is: Lobectomy, N.B: Bethesda for thyroid: 1,2,3 >>> Repeat. FNA 4>>>
Lobectomy. 5,6>>> near total thyroidectomy

13. Old man suddenly had vomited two episodes of bloody vomit, Physical Exam
shows no Tenderness over his abdomen or sternal area, he is stable, what is your
diagnosis?
A. Peptic ulcer
B. Mallory Weiss tear
C. Varicose veins
Answer is: B

3
14. Pt with dyspepsia for 3 weeks, medically free and can eat and drink solid and
liquid. What next management?
A. Urgent endoscopy
B. Refer to gastroenterology
Answer is: B, N,B: mostly Dx is functional dyspepsia.

15. Q about dysphagia to solids mainly and how to diagnose?


A. Chest X-ray
B. CT
C. Barium meal
D. NO manometry in the options
Answer is: C

16. (perforated viescus) Air under diaphragm. what is the diagnosis?


A. duodenal perforation
B. Intestinal obstruction
C. Biliary leakage
D. Septic hemorrhage
Answer is: A

17. A patient complained of Melena and epigastric pain diagnosed as peptic ulcer and
received PPI but not improved. Endoscopy showed multiple ulcers in the antrum.
What is the most accurate management?
A. Antrectomy
B. Partial gastrectomy
C. Total gastrectomy
Answer is: A

18. A patient with a small GIST tumor (less than 2cm), what is the most accurate
management?
A. Total gastrectomy
B. Partial gastrectomy
C. Observation
D. PPI
Answer is: C

19. 56 yrs old female complain of vague epigastric pain for last 6 months (long
scenario) (Diagnosis 5cm GIST)
A. Partial Gastrectomy
B. Total complete Gastrectomy
C. WLE
D. Not Remember
Answer is: C

20. colorectal Ca screening age


A. 50 years
Answer is: A

4
21. Most common cancer screen in asymptotic?
A. Colorectal Ca
Answer is: A

22. Old Man he suddenly had severe pain and vomiting, you did AXR and found Y
shaped Bowel, Per-rectal exam showed empty rectum what you should do?
(Nothing mentioned regarding his Stability)?
A. Fleet Enema
B. Colectomy with end stoma (I think Hartman procedure)
C. Sigmoidoscopy
D. US
Answer is: C, N.B: CT if no Do sigmoidoscopy or colonoscopy.

23. Coffee bean sign in x-ray?


A. sigmoid volvulus
B. cecal volvulus
Answer is: B

24. Case of obstruction symptoms, disk x-ray which showed air-fluid level What is the
most appropriate investigation?
A. CT
B. Diagnostic laparoscopy
C. Us
Answer is: A

25. Which type of cancer screening in asymptomatic populations?


A. colon
B. Ovarian
C. HCC
Answer is: A

26. Old male presented to ER with LLQ pain which is moderate in intensity and
increases by rebound tenderness. Diagnosis?
A. Colon cancer
B. Crohn’s disease
C. Diverticulosis
Answer is: C

27. 77 y old male presented with blood with stool & cachectic. Lab results show
marked anemia?
A. IBD
B. Anal fissure
C. Colon cancer
Answer is: C

5
28. Old male with a history of Crohn’s after abdominal surgery with about 1 month
developed vague abdominal pain & fever. By rectal examination, there was boggy
anterior. What to do?
A. ABT
B. Drainage
C. Metronidazole
D. Steroids
Answer is: B

29. old pt had open appendectomy after complicated rapture appendix, after 6 w he
came with sing of intestine obstruction. And sign of peritonitis. what is
appropriate step?
A. exploratory laparotomy
B. CT
C. Abx
Answer is: C, Then exploratory laparotomy

30. Post open appendectomy case with pain in wound site on examination u see pus
oozing from site of surgery what will u do next?
A. Percutaneous
B. US Abd
C. IV antibiotics
D. open surgery
Answer is: B

31. Case scenario appendicitis but not ruptured best investigation?


A. CT abd
B. US abd
C. Exploratory
Answer is: A, N.B: If child or female > US. If adult male > CT.

32. A patient has an appendiceal mass confirmed by us with no abscess MX?


A. Lap appendectomy now
B. Open appendectomy now
C. Lap appendectomy in 12 weeks
D. Open appendectomy in 12 weeks
Answer is: C

33. A patient with epigastric pain admitted to hospital, U/S shows gallstones with
dilated CBD. Now he is stable and labs are normal except high amylase 250. What
is the appropriate next step?
A. ERCP
B. CT abdomen
C. cholecystectomy now
D. cholecystectomy after 6 months
Answer is: A

6
34. A patient is recovered from acute pancreatitis episode presents after 6 weeks
with vomiting and epigastric fullness. CT showed cystic collection behind the
pancreas. what is the most likely diagnosis?
A. Pancreatic Pseudocyst
B. Pancreatic abscess
C. Pancreatic necrosis
Answer is: A

35. A case of acute cholecystitis. What is management?


A. Early lap cholecystectomy
B. Late lap cholecystectomy
C. Open cholecystectomy
Answer is: A

36. A 35-year-old woman undergoes an elective laparoscopic cholecystectomy for


symptomatic cholelithiasis. Which of the following wound classes best describes
her procedure? NEW20
A. Class I, Clean
B. Class II, Clean/contaminated
C. Class III, Contaminated
D. Class IV, Dirty
Answer is: B

37. Patient with jaundice, abdominal pain and on US u see stones and dilated CBD
vitals show fever only asking about diagnosis?
A. Ascending cholangitis
B. Choledocholithiasis Cholecystitis
Answer is: A, N.B: Charcot Triad (Fever, Pain, Jaundice)

38. A woman post lap cholecystectomy complaining of SOB and ascites confirmed by
US. Management?
A. ERCP
B. Abx
C. Exploration
D. Percutaneous drainage
Answer is: D

39. Q about old male 65 y with vague abdominal pain & jaundice & gallbladder NOT
tender although U.S. showed dilated CBD. Diagnosis?
A. Acute cholecystitis
B. Pancreas cancer
C. Stomach cancer
Answer is: B

7
40. Older male with asymptomatic femoral hernia What to do?
A. observe
B. open repair with mesh
Answer: B

41. An elderly patient with a symptomatic reducible hernia. what’s the treatment?
A. Simple repair
B. Laparoscopic repair
C. Open with mesh
Answer is: C

42. Old patient with a history of hernia repair 12 ago, now he has a complete bowel
obstruction and distended abdomen. what is the best next management?
A. Surgical reduction
B. Surgical resection
C. Observation
D. CT abdomen
Answer is: D

43. A 52-year-old female patient presented with pain, dragging sensation, and an
intermittent swelling in the abdominal region. After thorough evaluation she was
diagnosed with a ventral hernia. Which of the following is not a type of ventral
hernia? NEW20
A. Hiatal hernia
B. Epigastric hernia
C. Umbilical hernia
D. Incisional hernia
Answer is: A

44. 65 years old women went to the clinic. Incidentally diagnosed with a small
femoral hernia. What's your management?
A. Observation
B. Open with mesh
C. Open without mesh
Answer is: B

45. Old man came to you with Inguinal reducible hernia, and he was advised to do
hernia repair, and PE exam was normal he is asymptomatic what will you do?
A. No surgical treatment
B. Open repair with mesh
C. Laparoscopic repair (no mesh mentioned)
Answer is: A

8
46. Female with Painful Mass bulged lateral and inferior of pubic tubercle, Dx?
A. Femoral hernia
B. Indirect inguinal hernia
C. Direct inguinal hernia
Answer is: A

47. A man with hx of open hernia repair 10 y ago came with recurrent same inguinal
hernia reaching the neck of scrotum mx?
A. Open repair (didn’t say mesh)
B. No repair
C. Lap mesh repair
Answer is: C

48. Female 22 years old has a 2x2 cm on her hand since childhood. what to do?
A. follow-up
B. resection of mass
C. laser
Answer is: A, if for cosmetic > C.

49. An elderly patient in ICU has bed sores with exposed necrotic skin and
subcutaneous tissue. What is your management?
A. Debridement with a skin graft
B. Debridement with assisted vacuum
C. Debridement with primary closure
D. Debridement & Dressing with secondary closure
Answer is: B

50. A 43-year-old male underwent a major surgery. A few days later, his surgical
wound gets infected. What is the most likely source of infection? NEW20
A. The dressing tools and devices
B. The pressure dressing gauze
C. The staff’s hands during examination and dressing.
D. The patient’s visitors at visiting time
Answer is: C

51. Case of a patient complaining of abdominal pain, exam tenderness, and. The
pulsatile mass above Umbilicus What is the most appropriate investigation?
A. US
B. CTA
C. angiography
Answer is: A

9
52. Most appropriate diagnostic tool for abdominal pulsatile mass not diagnosed by
plain X-ray?
A. Abdominal CT
B. abdominal US
C. CT angiography
D. MR angiography
Answer is: B

53. Male with non-pitting L.L. edema and intact pulse what to do the best initial?
A. Venous duplex
B. CT Angio
C. Arterial duplex
D. NO lymphoscintigraphy in the options
Answer is: A

54. Trauma with tension pneumothorax GCS 8, what to do next?


A. Intubation
B. Needle thoracostomy
C. Chest x-ray
D. US
Answer is: B

55. stab wound below chest wall stable vitals what's most appropriate action?
A. exploration
B. CT scan
C. FAST
Answer is: C

56. Patient when knee examination, the tibia was forward anterior to the femur,
which ligaments are affected?
A. Anterior cruciate
B. Posterior cruciate
C. Lemnisci tear
Answer is: A

57. DVT prophylaxis in hip fracture?


A. Enoxaparin
B. Heparin Mechanical
C. Warfarin
Answer is: A

10
58. Trauma with isolated head trauma and coma for 5 days what’s the method for
nutrition in the first period
A. NGT
B. Central line
C. Peripheral line
D. Gastrostomy
Answer is: A

59. A 32-year-old male presents to the emergency department with a stab wound to
the abdomen. He is hemodynamically unstable. Which of the following is the best
next step in the management of this patient?
A. CT of the abdomen
B. MRI from head to toe
C. Abdominal radiograph
D. FAST and exploratory laparotomy
Answer is: D

60. A patient involved in RTA. He is stable, but there is a left leg swelling and
paresthesia between toes. X- ray showed fractured tibia. The pressure in the
posterior leg compartment is 35 mmHg. What is the best management?
A. internal fixation
B. external fixation with multiple fasciotomies
C. internal fixation with multiple fasciotomies
Answer is: B

61. A man after accident and resuscitation in small hospital u need to transfer to
another hospital after stabilization, it is 30 mins far. on xray u see fracture of 2-5
left ribs. no pneumothorax what will u do?
A. Intubate
B. call the other hospital to inform the surgeon on call
C. chest tube insertion
Answer is: B

62. 5 days post orthopedic surgery for fracture femur had sudden dyspnea and
confusion on examination shows rash on neck and on CXR bilateral lower lobe
infiltrates, cause?
A. Fat embolism
B. PE
C. Pneumonia
Answer is: A

63. Neck stab wound on (zone I) examination: subcutaneous emphysema, all vital
signs normal?
A. Neck exploration
B. Neck & chest CT
Answer is: B

11
64. 12 y with spleen 2 cm laceration and minimal fluid what is the mx?
A. Conservative
B. Surgery
Answer is: A, N.B: grade 1,2 > conservative, grade 3,4,5 > stable:
splenorrhaphy, unstable: splenectomy

65. Another child with spleen laceration but they mention the grade it was 3 asking
about the mx?
A. Splenectomy
B. Spleen preservation surgery (splenorrhaphy)
Answer is: B, by exclusion.

66. The best test to rule out cervical spine?


A. MRI cervical
B. AP X-ray of cervical
C. CT (without cervical)
D. clinical exam
Answer is: in trauma > C.

67. years old bot with femur fracture and 30% angulation. Management?
A. Closed reduction and hip spika
B. Closed reduction with traction
C. ORIF with nail
D. plate fixation
Answer is: according to age, but mostly he is 6 year > C.

68. A child with a case of blunt trauma, laceration of spleen 2 cm, Hb 11, HD stable
what is the treatment?
A. Two units of pRBCs
B. Nonoperative
C. Splenectomy
D. Splenic preservation survey
Answer is: B

69. management of open wound with fracture?


A. irrigation and debridement
Answer is: A, N.B: IV ABX is the first step in any open wound, then
debridement its definitive.

12
70. A 68-year-old female with diabetes mellitus, coronary artery disease,
fibromyalgia, and dyspepsia presents for follow-up. She has been taking
omeprazole (Prilosec) for 10 years. It was started during a hospitalization, and her
symptoms have returned with previous trials of discontinuation. Which one of the
following adverse events is this patient at risk for as a result of her omeprazole
use? NEW20
A. Hypermagnesemia
B. Urinary tract infections
C. Nephrolithiasis
D. Hip fractures
Answer is: D

71. A 25-days-old male infant is brought to the emergency department with


complaint of vomiting milk after feeding. Mother says that her son vomits
immediately after every breast-feeding, nonbilious and non-bloody emesis. The
symptoms started about 7 days ago, and that vomiting episodes have increased in
frequency. Which of the following is the most likely treatment for this patient?
A. Surgical correction with total gastrectomy
B. Surgical correction with partial gastrectomy
C. Surgical correction with pyloromyotomy
D. Conservative treatment with a special diet
Answer is: C

72. A woman was bathing her child and noticed a mass in his flank, which of the
following invx is most appropriate?
A. Abdominal radiography
B. Abdominal CT
C. Abdominal MRI
D. Abdominal radiography and ultrasonography
Answer is: D, Initial: US Most accurate: CT

73. Thenar muscle atrophy cause?


A. Median nerve
Answer is: A

74. Patient after varicose surgery i think Come with loss of sensation intact motor of
the foot which nerve injured?
A. Sciatic
B. Saphenous
Answer is: B, N.B: Sciatic > motor and sensory Saphenous > pure sensory
Varicose test? Duplex ultrasound

75. what DVT prophylaxis use in patient with recent eye surgery?
A. mechanical compression
Answer is: A

13
76. Nerve sensation of pinna go external ear?
A. Great auricular nerve
B. less occipital nerve
Answer is: A

77. An 82 years old male with very painful micturition and week urination. What is the
most appropriate management?
A. ABx for UTI
B. Foley catheter and ABx
C. Cystoscopy and TURP
Answer is: B

78. Part of urethra affected by surgical trauma in males?


A. membranous
B. penile
C. prostatic
D. Bulbar
Answer is: D

79. Pt with pelvic Fx and bleeding per rectum >> urethrogram > retroperitoneal
urethra injury Mx?
A. Folly's cath
B. Supra pubic cystostomy
C. Laparoscopic repair
Answer is: B

80. pt. with heart failure and admitted for surgery for some disease and connected
to IV fluid, post-op 2 days later complained of SOB and bilateral basal.
Crepitation, how could this be prevented?
A. IV Furosemide immediate post-op
B. monitoring IV fluids daily
Answer is: B

81. Post appendectomy female came with LR abdomen mild tenderness Ex Normal
By CT there is 2*2 collection in Retrocecal?
A. Exploring laparotomy
B. percutaneous drainage symptomatic
C. laparoscopic
D. conservative with Antibiotic
Answer is: D

82. male came with multiple episodes of forceful vomiting He has abdominal pain,
dyspnea, and Subcutaneous emphysema in the chest What is the diagnosis?
A. Esophagitis
B. Perforated duodenal ulcer
C. Boerhaave syndrome
Answer is: C

14
83. 40 years old man underwent open hernia relain and 2 weeks later presented
with tenderness at the site of hernia repair with mesh and severe
parasthesia/numbness/tingling around his thigh that went down his leg
management?
A. remove mesh staples.
B. Neurectomy and mesh removal.
C. Non-steroidal anti-inflammatory drugs
D. Nerve block
Answer is: C

84. Invasive intraductal papilloma. The most appropriate management is?


A. Wide local incision
Answer is: A

85. Female 30s asking for cosmetic treatment of varicosity in her thigh,
asymptomatic, no other varicosity, how would u investigate?
A. Venous Duplex
B. CT venography
C. No need for further investigation
Answer is: A

86. colon cancer surgery after that he had 7ml/h urine output for 8 hours, blood
pressure and heart rate was normal what is the best management?
A. 500 NS challenge
B. diuretic.
C. Inotropes
Answer is: A

87. Case of internal hemorrhoid grade 4, what is the most appropriate treatment?
A. open hemorrhoidectomy
B. rubber band ligation
Answer is: A

88. Adult pt feels groin pain especially with exercise. Exam of groin and genitals
normal, next step?
A. Assess after 2 weeks
B. Us
C. Ct
D. MRI
Answer is: D

89. Patient with GIST in the body of stomach, size 5 What to do?
A. WLE
B. Observe
C. Chemo
Answer is: A

15
90. Case of angiodysplasia in left colon: Large amount- pt is hypotensive and
tachycardia?
A. Endoscopic laser
B. Conservative
C. Angioembolization
D. left hemicolectomy
Answer is: C

91. sharp, shooting pain of neck radiated to shoulder and jaw, what is the
diagnosis?
A. cervical disk prolapses
B. multiple sclerosis
C. rheumatic myalgia
Answer is: A

92. A 62 YO man medically free c/o 1-day hx of 4 episodes of hematochezia no abd


pain, constipation and wt loss. Dx?
A. Diverticulosis
B. Cancer
C. Ischemic colitis
D. Internal piles
Answer is: A

93. Case of RUQ pain and tenderness jaundice, fever elevated total bilirubin not
mention direct or indirect on US multiple GB stones no peri cystic fluid CBD is
dilated 1cm?
A. Ascending cholangitis
B. Acute cholecystitis
C. Choledocholithiasis
D. Acute pancreatitis
Answer is: A

94. 66 Y.O Pt heavy smoker with symptoms of GERD, upper endoscopic


examination showed squamous cell with high-grade dysplasia, what is the
most appropriate next step?
A. Council him about smoking cessation.
B. Refer for possible esophagectomy.
C. PPI and follow up after 6 months.
D. Follow up after 3 months.
Answer is: B

95. Case of melanoma, male with skin ulcer severe pigmentation indurated what is
the treatment?
A. biopsy
B. excision
Answer is: B

16
96. 26 Y.O female diagnosed with Crohn's disease. On endoscopy she had 1cm
stricture in the terminal ilium. What is the most appropriate next step?
A. Strictureplasty
B. Small bowel resection
C. Resection with ileostomy
D. IV antibiotics
Answer is: C

97. A female presented to the clinic with non-tender lump in the right breast her
mother was diagnosed with cancer, Biopsy was taken shows atypical duct
hyperplasia?
A. Chemoprevention
B. Simple mastectomy
C. Follow up in 6 months
D. wide local excision
Answer is: D

98. Diabetic patients came with toe Ingrown nail What is the appropriate next
step?
A. Culture and sensitivity
B. Assessment of another toe
C. Us
D. check Peripheral pulse
Answer is: D

99. Dm pt with unilateral leg erythema Increases in dependent position, Cold,


Femoral pulse is present, distal pulse can be palpated No tenderness, no
swelling, no fever Otherwise unremarkable Dx?
A. Cellulitis
B. Arterial insufficiency
C. Superficial thrombophlebitis
Answer is: C

100. 28 male healthy, hx 2 month of abd pain with 2 bloody stool,


proctoscope done with numerous polyps covered the linings and
multiple biopsy taken, no details in hs of family hx or sexual.. Dx?
A. Familial polypoid
B. UC
C. Diverticulosis coli
D. Human papillomavirus polyp
Answer is: A

17
101. Colorectal surgeon performing a low anterior resection for CRC and the pelvis
won’t stop bleeding, so he consults a vascular surgeon and he does?
A. Heavy packing of pelvis
B. Arteriography intra operatively
C. Infraceliac clamping
D. Infrarenal clamping
Answer is: A

102. patient who came with nausea and vomiting and found out to have
appendicitis what is the pathophysiology in this case?
A. Low cardiac index
B. Vasoconstriction
Answer is: B

103. Typical case. patient positive 5 HIAA?


A. Carcinoid tumor
Answer is: A

104. Burn case black soot over nostrils and mouth 40% carboxyhemoglobin. (Carbon
monoxide toxicity)?
A. Hyperbaric oxygen.
B. Intubation and ventilation with 100% 02
C. Carbonic anhydrase inhibitors.
Answer is: B

105. Patient was hit with wood 5 days ago, now he comes with severe RLQ pain. On
examination there is a small opening with pus discharge, put when tried to
extend his thigh there was severe pain and you couldn't move it. What to do?
A. CT
B. wound drainage
C. antibiotics
D. culture from discharge
Answer is: A

106. Patient 56 c/ o sudden RLQ pain after lifting heavy object; and mass in RLQ;
Cough negative; abd muscle tense even with clenching. He is on
anticoagulation due to A fib and INR high. What the most appropriate
management?
A. Angioembolization
B. rest and analgesic
C. FFP
Answer is: stable > B, unstable > C. here answer is C because high INR.

18
107. 40-Y/O female with 12-year history of varicosities, complaining of LL edema
and heaviness when standing. Distal pulses are intact, and there is bilateral
varicose in the great saphenous vein territory. What is your management?
A. Sclerotherapy
B. Endovenous laser ablation
Answer is: B

108. 58-year-old patient has an appendiceal mass with no abscess then treated
by non-operative, what is next appropriate step?
A. colonoscopy in 6 weeks
B. Lap appendectomy in 12 weeks
C. Open appendectomy in 12 weeks
Answer is: A

109. Diabetic patient with pseudo hyper epithelialization in situ, what you should
do?
A. Amputate toe
B. ulcer Debridement
C. follow up
D. Repeat biopsy
Answer is: B

110. Trauma patient with a wound on his thigh, subcutaneous fat is lost and
vasculature underneath is exposed what provides the best management?
A. Debridement with primary closure
B. Primary repair
C. Debridement with secondary graft
D. Debridement with vacuum assisted closure
Answer is: C

111. adrenal adenoma discovered accidentally in CT. Most common adrenal


mass discovered accidentally?
A. sub clinical Cushing
B. Conn’s
C. cancer
D. nonfunctional adenoma
Answer is: D

112. Patient female within 50-74 y, when to do mammogram?


A. Every 2 years
Answer is: A

19
113. years old, breast mass (10 * 12) cm malignant phyllodes tumor, no auxiliary LN,
management?
A. Wide local excision
B. hormonal or biologic agent
C. Simple mastectomy
Answer is: C, N.B: Phyllode treatment: Benign: WLE (Wide local excision),
Malignant: Simple mastectomy

114. BIRADS stage 5, MGR?


A. Follow up
B. surgery
C. Core biopsy
Answer is: C

115. year old with breast mass that is mobile and small and well localized, what is
Diagnosis?
A. Fibroadenoma
Answer is: A

116. 47-year-old with breast mass that was small and enlarging with time, thin skin
but no inflammation, what is the diagnosis?
A. Phyllode tumor
Answer is: A

117. management of phyllode tumor?


A. WLE
B. simple mastectomy
Answer is: A

118. Which of these are associated with severe eye symptoms in thyrotoxic pt?
A. Male gender.
B. Resistance to thiouracil drugs.
C. I don’t remember the rest
D. Smoking
Answer is: D

119. Malignancy associated with Hashimoto thyroiditis?


A. Papillary.
B. Nodular.
C. Medullary.
D. Lymphoma.
Answer is: D

120. Bethesda Stage 4 management?


A. lobectomy
Answer is: A

20
121. Patient is known to have chronic gastric ulcers at pylorus, presented with
vomiting and abdominal pain. UGI Endoscopy: narrowing at the pylorus end due
to chronic ulcers, what abnormality are likely present?
A. increase urinary potassium
B. metabolic acidosis
C. hypokalemia
Answer is: C

122. a pt with hx of H. Pylori infection along with MALToma?


A. Eradicate H. Pylori
Answer is: A

123. Anterior duodenal perforation treatment?


A. Graham omental patch
Answer is: A

124. Anal fissure with skin tag, management?


A. Internal lateral sphincterotomy
B. Curettage of fissure with removal of the skin tag
Answer is: A

125. year-old female with history of constipation and bleeding post defecation, also
shows in examination skin tag what is the management?
A. anterior sphienectrotmy
B. lateral sphienectrotmy
C. laser repair and skin Tag removal with laser
D. do nothing
Answer is: B

126. sclerosing patient with history of rectal bleeding, anoscopy show swelling at 3,7
o'clock, sclerosing therapy is planned. what is the most appropriate to do
sclerosing therapy? NEW20
A. internal hemorrhoids
Answer is: A

127. Osteoporosis is one of extraintestinal manifestations of Crohn’s disease, which of


the following will increase its risk if associated with CD?
A. age20-40
B. premenopausal
C. long term steroids
D. extensive bowel involvement
Answer is: C

21
128. Post open appendectomy 7 days came with serous discharge multiple time. On
examination vitally stable, abdomen soft tenderness at site of incision with clear
discharge. Management?
A. abdominal CT
B. dressing (regular or daily)
C. percutaneous drainage
Answer is: B

129. Case of appendicitis but not ruptured best investigation?


A. CT abd
B. US abd
C. Exploratory
Answer is: A, N.B: If child or female > US If adult male > CT

130. something morbidly obese male, how to decide best reduction surgery he will
have?
A. Barium enema
B. CT abdomen
C. ultrasound abdomen
D. GI endoscopy
Answer is: D

131. post sleeve gastrectomy pt has change bowel habit, anxiety, what is treatment?
A. reassurance
Answer is: A

132. CT finding of Hepatic mass: encashment of the borders, with progressive filling?
A. hepatoma
B. Hemangioma
C. Mets
D. Hydatid cyst
Answer is: B

133. Patient diagnosed 10 years ago with gallstone, Air in the biliary system, what is
the cause?
A. Cholecystitis
B. Gallbladder ileus
Answer is: B, air in biliary cause by necrotizing infection or fistula. If there
intestinal obstruction > B.

134. Major burn, referred to surgery for 24 hours symptoms of cholecystitis,


management?
A. cholecystostomy tube
B. Open cholecystectomy
Answer is: A

22
135. Patient had laparoscopic cholecystectomy then developed ascites. Next
management?
A. ERCP.
B. diagnostic laparoscopy
C. exploratory laparotomy
D. percutaneous drainage
Answer is: D

136. Patient came with RUQ pain, us finding thickening gallbladder wall and pericystic
fluid and stones What is the Best intervention?
A. percutanous drainage.
B. laparoscopic cholecystectomy
Answer is: B

137. Patient with jaundice, abd pain and on US u see stones and dilated CBD vitals
show fever only asking about diagnosis?
A. ascending cholangitis.
B. choledocholithiasis
C. cholecystitis
Answer is: A

138. pt with necrotizing pancreatitis, drain was inserted and the pt improved what is
the metabolic effect?
A. Insulin resistance
B. Hypoglycemia
C. Inhibition of gluconeogenesis
D. Lipolysis
Answer is: D

139. 40 years old woman with no pain but you noticed jaundice. She has high direct
bilirubin and high ALT what is your diagnosis?
A. Carcinoma in head of pancreas
Answer is: A

140. Patient presented with upper abdominal pain she has Hydatid cyst 4 cm Us:
Found daughter cyst … What is the appropriate management?
A. Albendazole
B. Surgical Deroofing
C. Percutaneous drainage
Answer is: B

141. jaundice with us show stone in GB and in CBD, tx?


A. Laparoscopic cholecystectomy
B. ERCP
Answer is: B

23
142. old male 65 y with vague abdominal pain & jaundice & gall bladder NOT tender
although U.S. showed dilated CBD. Diagnosis?
A. acute cholecystitis
B. pancreas caner
C. stomach cancer
Answer is: B

143. Elderly Patient admitted to ICU with acute Myocardial infarction and developed
RUQ pain and tenderness, management?
A. Do elective surgery after 6 weeks.
B. surgery after 6 months
Answer is: B

144. After ERCP surgical emphysema?


A. Duodenal perforation
Answer is: A

145. Patient came with cholecystitis symptoms for 1 day, how will you manage?
A. early lap chole
B. 2-3m lap chole
C. ERCP
Answer is: A

146. Elderly presenting with a picture of ascending cholangitis. What's the best
management?
A. ERCP
B. Lap chole
C. Percutaneous cholecystostomy
Answer is: A

147. Case of pancreatic pseudocyst has follow up during last 3 month increasing in
size what is management?
A. Follow up
B. Internal drainage
C. percutaneous drainage
Answer is: B

148. Most common organ damage in ERCP?


A. Duodenum
Answer is: A

149. Hernia with signs of Intestinal obstruction obstructed?


A. Incarcerated
B. Strangulated
C. irreducible
Answer is: obstructed hernia, if not there > B.

24
150. 5 y/o child. Bilateral inguinal hernia with swelling in inguinal scrotal line?
A. Herniotomy
B. Laparoscopic hernia repair with mesh
C. linschoten hernia repair
Answer is: A

151. diabetic patients with unhealed ulcer for 5 years and biopsy showed pseudo
epitheliomatous hyperplasia?
A. Debridement
Answer is: A

152. surgeon who did resection for low anterior part of colorectal & he call for
vascular surgeon & there was heavy bleeding which he try to encounter with
packing but he still bleeding what he should do?
A. do on table Angio
B. heavy packing
C. Infra celiac clamp
Answer is: B

153. Hard sign of vascular injury?


A. Palpable thrill
B. Weak pulse
C. Changed the color of the limb
Answer is: A

154. patient had bypass surgery and has heart rate of 120 and bp 90/60 and
Increased peripheral resistance, what type of shock is that?
A. cariogenic
B. hypovolemic
C. septic
Answer is: A

155. pt with pleural effusion and doctor want to do Thoracentesis where?


A. 4
B. 5
C. 7
D. 9
Answer is: D

156. MVA case, tachycardia, hypotension, low pulmonary wedge pressure, high
Stroke volume what type of shock?
A. Hypovolemic
B. Cardiogenic
C. Septic
Answer is: C, N.B: high Stroke volume.

25
157. Trauma patient, with typical signs and symptoms of tension pneumothorax, his
GCS is 8, what is your next step in management?
A. intubation
B. Needle thoracostomy
Answer is: B

158. Patient was hit on abdomen 4 days ago. Found to have small wound with semi-
purulent discharge, what’s the most appropriate management?
A. CT of abdomen
B. Send for laparotomy
C. Culture the discharge
Answer is: A

159. Pt after hit by car presents with SOB; OE: pt slightly confused; chest: decrease air
entry and hyper-resonant CXR: hyperlucency on one side What is the most
appropriate next step??
A. intubate
B. needle decompression
Answer is: B

160. what to give initially after intubation on ER presentation? NEW20


A. Diazepam
Answer is: A

161. patient with 20% back burn, what indicated a good prognosis?
A. Urine output 0.9/kg/hr
Answer is: A

162. A man after accident and resuscitation in small hospital u need to transfer to
another hospital after stabilization, it is 30 mins far. on x-ray u see fracture of 2-5
left ribs. no pneumothorax what will u do?
A. intubate
B. call the other hospital to inform the surgeon on call
C. chest tube insertion
Answer is: B

163. Neck stab wound on (zone I) examination: subcutaneous emphysema, all vital
signs normal?
A. neck exploration
B. neck & chest CT
Answer is: B

26
164. A man presents to the emergency department after suffering from an
automobile accident. The patient is conscious, alert, and oriented. If the patient
also presents with cold and clammy extremities, what type of shock is this?
A. Cardiogenic
B. Hypovolemic
C. Neurogenic
D. Irreversible
Answer is: B

165. A patient presents to the emergency department 30 minutes after being


involved in a traffic accident. Physical exam reveals elevated JVP. Auscultation of
the chest reveals normal air entry bilaterally, with muffled heart sounds. Which of
the following is the most likely diagnosis?
A. Pneumothorax
B. Hemothorax
C. Cardiac tamponade
D. Acute coronary syndrome
Answer is: C

166. A patient presents to the emergency department with paralysis of all four
extremities after being involved in an automobile accident. The patients'
extremities are warm and pink. Which of the following is the most likely
diagnosis?
A. Hemorrhagic
B. Irreversible
C. Neurogenic
D. Cardiogenic
Answer is: C

167. Tingling in ring al little fingers with positive elevation test?


A. Carpal tunnel syndrome
B. ulnar artery thrombosis
C. Thoracic outlet obstruction
Answer is: C

168. Patient with drop foot, he lost the sensation in the webspace between 1st & 2nd
metatarsal joints. What is the damaged nerve?
A. common peroneal nerve
B. Deep Peroneal nerve
C. Tibial nerve
D. Femoral nerve
Answer is: B

27
169. A 33-year-old male complaining of pain and numbness over the left thumb and
index, loss of sensation over the upper 1/3 of the dorsum of the left hand. Which
of the following investigations is best to confirm the diagnosis? NEW20
A. Ultrasonography
B. MRI
C. Nerve conduction studies
D. Computed tomography
Answer is: C

170. Boy with midline cervical or just below hyoid bone mass that move with
swallowing What is diagnosis? NEW20
A. Cystic hygroma
B. Thyroglossal cyst
C. Submental lymphadenopathy
Answer is: B

171. Child came with testicular swelling wasn’t painful, positive transillumination
normal sensations in preanal area?
A. Testicular torsion
B. Hematocele
C. Hydrocele
D. Indirect hernia
Answer is: C

172. Elderly with urinary catheter, with positive culture?


A. Check urine catheter daily
Answer is: A

173. years old child found to have one testis in the scrotum and the other in the
inguinal area what to do? NEW20
A. orchiopexy
Answer is: A

174. years old patient presented with vomiting, RLQ pain, and rebound tenderness.
No history of fever or leukocytosis. An X-ray showed stone in the ureter. What is
your management?
A. urgent urology consultation
B. appendectomy
C. CT without contrast
D. U/S for gallbladder
Answer is: C

28
175. patient present with presentation of cholangitis epigastric pain I think (clinically
not very typical but from the labs it looks like cholangitis). NO vitals provided. His
labs show High ALP, direct bilirubin. What’s the most important initial
management?
A. IVF
B. ABx
C. abdominal US
D. ERCP
Answer is: A

176. How to find appendix?


A. follow taenia coli
Answer is: A

177. Patient after pancreatitis episode develops upper GI bleeding picture, scope was
done, gastric fundus bleeding was found, sclerotherapy done Duplex ultrasound
showed: splenic vein thrombosis with patent portal vein what’s is your
management?
A. Splenectomy
B. Portal-systemic shunt
Answer is: A

178. 60 years old man had an Open surgery and he noticed 20ml fluid come from
midline abdomen what’s your appropriate management?
A. Dressing
B. Antibiotics
C. Ct abdomen
D. Wound exploration
Answer is: A

179. A 9-year-old girl come to ER complaining of fatigue and lethargy physical


examinations she is pale she has hepatomegaly and splenomegaly 6 cm below
coastal area hemoglobin low 50 sickle cell positive she had seam attack 2 times
before what’s your best management?
A. Regular blood transfusions
B. Corticosteroids
C. Hydroxyurea
D. Splenectomy
Answer is: D

180. Man come to the ER for poly Truma and he was intubated after resection
admitted to the ICU what is the best approach to know cervical spine injury?
A. MRI neck
B. Ap and lateral x-ray neck
C. Clinical judgment
D. Ct neck
Answer is: D

29
181. CI of liver transplant?
A. Active alcohol
Answer is: A

182. known case of crohns came with multiple strictures in bilary tract?
A. primary sclerosing cholangitis
Answer is: A

183. UC what increase risk of colorectal cancer?


A. UC for 3 years duration
B. Primary sclerotic cholangitis
C. Mild inflammation
Answer is: B

184. after open inguinal hernia repair came with decrease size of one testis?
A. Ischemic orchitis secondary to pampiniform plexus thrombosis
Answer is: A

185. Which structure pass through inguinal ring?


Answer is: Ilioinguinal nerve

186. Patient bleeding from angiodysplasia left side, Mx?


A. Left hemicolectomy
B. Angio-embolization
C. Endoscopic laser ablation
Answer is: Stable > C, Unstable > B.

187. pancreatitis case managed conservatively at the day 3 the pt improving clinically
and laboratory. Everything normal except Amylase 250. US showed>> Gallstone at
the gall bladder with dilated extra hepatic ducts (nothing was mentioned about
CBD). What is the appropriate management?
A. lap chole before discharge
B. elective lap chole
C. ERCP
Answer is: A

188. melanoma with no lymph nodes involved?


A. excision
Answer is: A

30
189. Elderly male pt, resident of nursing home with hearing aid presented with
cerumen impaction. Examination revealed bilateral Rene test + and Weber
lateralized to the right, what is the most likely dx?
A. conductive hearing loss more in the right ear
B. presbycusis is more the right ear
C. chronic otitis media in the left
D. Acoustic neuroma in the right
Answer is: A

190. give vesicoureteral barium and there’s dilatation in one of ureter & bladder &
pelvic?
A. cystourethral reflux
Answer is: A

191. 24 years old with hx of appendectomy 5 years ago present with abdominal pain,
distention, vomiting for 3 days, CT scan show obstruction signs and peritonitis
what is contraindicated in this pt?
A. Propofol
B. Ketamine
C. Sevoflurane
D. Nitrous oxide
Answer is: D

192. after accident found to have air under the diaphragm & spleen laceration &
thoracic aortic perforation (and hypotension) what first thing to do?
A. Thoracotomy
B. Exploratory Laparotomy
Answer is: B

193. Anterior thigh stab & severe bleeding, “labs show little low hg in 80 & NO vital
signs” what’s the initial steps? NO direct pressure in the options
A. ringer lactate IV Fluid
B. Tourniquet
Answer is: A

194. Patient with a stab wound in the right thigh and massively bleeding brought to
the emergency department unconscious and O/E there is active bleeding. What is
the most important next step to do?
A. Ringer lactate iv fluid
B. Blood transfusion
C. Tourniquet on the thigh
D. Orotracheal intubation
Answer is: D

31
195. Pt presented with stabbed wound after wound exploration you found anterior
abdominal fascia penetration, (his vitals were stable) what's your next step?
A. CT abdomen
B. MRI abdomen
C. Exploratory laparotomy
D. Diagnostic laparoscopy
Answer is: D

196. Child with stab wound in the lower right chest, free fluid in the abdomen, what
the most appropriate thing to do?
A. Thoracotomy
B. Tube thoracostomy
C. Exploratory laparotomy
Answer is: C

197. female get high energy accident (high velocity), with seat belt sign, on X ray have
chance fracture, what will associated with this fracture?
A. Duodenal perforation
B. Gastric perforation
C. Jejunum perforation
D. Vena cava perforation
Answer is: A

198. A 30 years old male present to E.R. complaining of visual deterioration for 3 days
of Rt. Eye followed by light perception, the least cause is? NEW20
A. Retinal detachment.
B. Central retinal arterial embolism.
C. Vitreous hemorrhage.
D. Retro-orbital neuritis.
E. Retinitis pigmentation.
Answer is: E

199. Symptoms of Cardiac tamponade ask about management?


A. Pericardiocentesis
Answer is: A

200. In a patient post mastectomy, they do for for her reconstruction from rectus
muscle, what is the vessels to be injured or effected? NEW20
A. Inferior Epigastric artery
B. Superior Epigastric artery
C. Intercostal artery
D. Internal thoracic artery
Answer is: B

32
201. 35YO with dyspnea during sleep, dysphagia for 4m, k/c of asthma, image shows
midline mass compress over trachea?
A. Goiter
B. Thymoma
C. Lymphoma
D. Lung cancer
Answer is: B, if there was neck mass > A.

202. Femur fracture the developed dyspnoea + x ray shows diffuse finding + rash
what is the diagnosis?
A. PE
B. Fat embolism
Answer is: B

203. Patient known to have hashimito disease present with a enlargement of thyroid
with heard bruit and US show extension of thyroid what is the cancer?
A. follicular
B. Medullary
C. Lymphoma
D. Other option
Answer is: C

204. Strong factor in Graves?


A. Male
B. Smoking
C. Increased free t3 & t4.
Answer is: B

205. What indicate poor prognosis in acute pancreatitis with elevated liver and
pancreatic enzymes and haematocrits?
A. Lipase >600
B. Amylase >500
C. Haematocrit >44%
D. Forgot the fourth on)
Answer is: C

206. Color of discharge in intraductal papilloma?


A. Red
B. Green
C. Black
D. White
Answer is: A

33
207. Thyroid nodule 2 cm with normal TSH and free t3 and t4?
A. US
B. CT neck
C. FNA
D. Thyroid scan.
Answer is: A

208. GIST 3*4cm + no LN mx?


A. Wide local resection
Answer is: A

209. Pt heavy smoker with symptoms of GERD, upper endoscopic examination show’s
squamous cell with high-grade dysplasia, next step in management:
A. Council smoking cessation
B. Endoscopic mucosal resection
Answer is: B

210. Female post gastric bypass surgery (i think 8 weeks) complaining of recurrent
vomiting and bad mood, all labs normal, CT scan normal, what’s your next step?
A. exploratory laparotomy
B. reassure
C. Psychiatric consultation
Answer is: B

211. Case of hydatid cyst with daughter cysts, what is the management
A. deroofing surgery
B. Percutaneous drainage
C. IV Alparanazole alone
Answer: A

212. Old patient known to have hepatitis C, present with weight loss, fatigue... US
report show mass (forget description), what’s is the diagnosis?
A. hepatocellular carcinoma
B. Hepatocellular adenoma
Answer is: A

213. Patient present with multiple vomiting then he vomitus blood “ case of Mallory
wiss tear” what investigation U will do ?
A. Upper GI endoscopy
Answer is: A

214. male patient with vomiting after food poisoning 2 day ago the he developed
blood vomiting with epigastric pain?
A. Mallory Weiss
B. Peptic ulcer
Answer is: A

34
215. 26y/o female with multiple masses felt in the left breast, gets tender around
period. U/S revealed multiple small cystic lesions and two solid masses?
A. Fibroadenoma
B. Fibrocystic
C. Breast cancer
Answer is: B

216. Adult case of intussusception, asking about management?


A. barium enema
B. surgical resection
C. surgical reduction
Answer is: B

217. Girl child complain of sever abd pain, red jelly like stool intussusception What is
the most appreciate Mx?
Answer is: Barium enema

218. patient know case of crohn’s disease came with abdominal pain there was intra
large intra-abdominal collection and iliujujenl fistula how to manage?
A. Percutaneous drainage
B. Open drainage with fistula repair
Answer is: A

219. Baby with poor wight gain and chronic diarrhea and there was stricture on
terminal ileum what is the dx?
Answer is: Crohn’s disease

220. Pain with defecation and you found fistula w/ skin tag what’s management?
Answer is: bad recall: mostly they mean fissure so answer will be > internal
Lateral sphincterotomy.

221. Old pt after rectal surgery he is not doing well after in the recovery he starts to
have leg pain he developed DVT from the popliteal to the femoral?
A. Enoxaparin
B. Heparin
C. Warfarin
D. IVC
Answer is: B

222. a patient was hospitalized complaining of unilateral leg swelling and pain, on
examination femoral pulse is intact but peripheral pulses are not intact, what is
the next step to do?
A. heparin
B. morphine
C. CT angiography
D. venous duplex
Answer is: A

35
223. A 62-year-old man with no significant medical history presented to Emergency
room with 1- day history of 4 haematochezia, He denies history of abdominal
pain, constipation or weight loos, Blood pressure 90/50 mmHg Heart rate 112
/min. Which of the following is the most likely diagnosis?
A. Caecal cancer
B. diverticulosis
C. Internal piles
D. Ischaemic colitis
Answer is: B

224. Scar post-surgery increases in size Extend beyond border?


Answer is: keloid

225. 60years old man had an Open surgery and he noticed 20ml fluid come from
midline abdomen what’s your appropriate management?
A. Dressing
B. Antibiotics
C. Ct abdomen
D. Wound exploration
Answer is: A

226. MI and peri collection no aberrant stone in biliary tenderness in palpation fever.
What to do?
A. Emergency cholecystectomy
B. Percutation drainage
C. ERCP
Answer is: B

227. A patient with left lower quadrant pain came to the ER, on abdominal CT you see
an inflamed appendix along with a large abscess extending to the abdominal wall,
what is your mx?
A. Percutaneous drainage
B. Laparoscopic appendectomy
C. Open appendectomy with drainage
D. Laparoscopic appendectomy with drainage
Answer is: A

228. Pancreatic pseudocyst with signs of infection best management?


A. percutaneous drainage
B. surgical resection
C. Internal drainage
Answer is: A

36
229. A patient with low tsh , normal t4 t3, he has no goiter and us revealed normal
thyroid, lymphadenopathy , they did aspiration to small mass in his lateral neck
and it revealed follicular thyroid cells , no other symptoms, dx?
A. Ectopic thyroid
B. Thyroid carcinoma
Answer is: B

230. pt with enlarged lymph nodes and normal thyroid FNA of LN showed normal
follicular thyroid tissue?
Answer is: thyroid carcinoma

231. patient with cervical lymph node and excisional biopsy shows normal follicular
cell and thyroid scan I don't remember but was normal?
A. reassurance
B. follow up after 3 months
C. surgical referral
D. excisional lymph node something
Answer is: C

232. Case scenario Above anal verge 2cm cauliflower appearance on protocope -
friable to tough?
A. Anal cancer
B. Rectal cancer
C. Something polyps
D. condyloma acuminata
Answer is: B

233. Old pt. C/O fatigue, on & off stool mixed with blood. O/E 2-degree hemorrhoid,
and low Hb Dx?
A. rectal cancer
B. Chronic bleeding hemorrhoid
Answer is: A

234. 15 yo came with groin pain and lower flank pain for (4h) or (8h) P/E: the testes
are slightly ascended and tender in touch and no fever. And Redness. What the
appropriate next step?
A. Doppler US
B. Expiratory surgery
C. Abx
Answer is: B

235. Treatment of medullary thyroid cancer?


A. Total thyroidectomy
B. Hemi lobectomy
C. Radiation
Answer is: A

37
236. Post appendectomy patient presented with seroma collection, what the
appropriate management?
A. Regular dressing
B. Wound Exploration
C. US guided drainage
Answer is: A

237. Case of nephrotic syndrome complaining of abdominal pain, diarrhea and


vomiting, with rebound tenderness, what most likely diagnosis?
A. Gastritis
B. Peritonitis
Answer is: B

238. Obese patient diagnosed with hiatel hernia, which of the following is best
operation for wight reduction?
A. Roux en Y
B. Sleeve gastrectomy
Answer is: A

239. 65 yr presented with dysuria and cloudy urine associated with bubbling with hx
of left iliac fossa pain for 2 years. Sigmoidoscopy: Narrowed lumen with normal
mucosa. Cystoscopy: Redness in the dome of bladder What is the most likely
diagnosis?
A. ischemic colitis
B. diverticular disease
C. transitional bladder cancer
D. squames bladder cancer
Answer is: B

240. Trauma patient has basal skull fracture that includes the jugular foramen what
will the patient have?
A. ipsilateral vocal cord paralysis
B. Paralysis of the muscles of mastication
C. Can’t abduct eye
Answer is: A

241. child fall down of stairs and hit her head, blood coming out from ear, perforation
of tympanic membrane?
A. Subachanoid Haemorrhage
B. Ventricles haemorrhage
C. Basel skull fracture
Answer is: C

38
242. 45-year-old female developed hernia located inferior and lateral to pubic
tubercle. What is the Type?
A. Femoral hernia
B. Obturator hernia
Answer is: A

243. Treatment of Phyllodes malignancy without lymph node involvement?


A. WLE
B. Mastectomy
C. Radiotherapy
Answer is: B

244. Female with benign phyllodes tumor asking about management?


A. Wide local excision
B. Wide local excision with radio
C. Mastectomy
D. Reassurance
Answer is: A

245. Multigravida pt came with midline mass, apprears when leaning forward.
Negative cough impulse. Intact anterior abdominal fascia. What's the most likely
diagnosis?
A. Spigelian hernia
B. Incisional hernia.
C. Divercation of recti.
D. Transversalis muscle weakness.
Answer is: C

246. Female pt with RA, using methotrexate. Labs were done 4 months ago and it
showed high LFT. US showed 5cm hypodense lesion on the upper border of liver.
What’s the most appropriate next step in management?
A. Stop methotrexate.
B. Resection of liver.
Answer is: A

247. Female 16 week pregnant, came with right sided abdominal pain (RLQ)
associated with nausea and vomiting. The doctor ruled out all pregnancy related
abdominal pain. What's the most likely diagnosis?
A. Gastritis
B. Pancreatitis
C. Cholecystitis
D. Appendicitis
Answer is: D

39
248. Thigh wound, came with active bleeding. What's the next step?
A. Infusion of ringer lactate.
B. Tourniquet around the wound.
Answer is: pressure on wound, if not there > A.

249. Patient developed sudden onset of headache, described as worst headache of


his life, Dx?
Answer is: Sub arachinodal hemorrhage.

250. Pt with hx of IHD with no fever, low CO, low wedge pressure and normal Rt
ventricular pressure. What’s the type of shock?
A. Septic
B. Cardiogenic
C. Anaphylactic
D. Hypovolemic
Answer is: D

251. Pt with bleeding. INR 2.1 what to give?


A. Vitamin K
B. FFP
C. Blood transfusion
Answer is: B, if there was Vit K and FFP both is more accurate.

252. A fib INR 3.9, all normal?


A. vit k
B. plasma
C. vit k and plasma
Answer is: A, if there was bleeding > C, if no bleeding > A.

253. patient with afib on warfarin, had a trauma and was diagnosed with subdural
hematoma that required evacuation, INR=3.9, next step?
A. vit k
B. fresh frozen plasma
C. vit k and fresh frozen plasma
Answer is: C

254. Pt diagnosed with thigh sarcoma. What the staging method?


A. Bone scan
B. CT of the chest
Answer is: B

255. diagnostic for sarcoma?


A. Incisional
B. Excisional
C. Core needle
D. Forget maybe aspiration
Answer is: C

40
256. Elderly with constipation and left lower quadrant fullness. What's the diagnose?
A. Constipation
B. Ureteric stone
C. Urinary tract infection
Answer is: A

257. Pt underwent mastectomy, complaining of numbness in medial aspect of the


arm. What's the nerve injured?
A. Long thoracic nerve
B. intercostobrachial nerve
Answer is: B

258. Male adult with perianal swelling and discharge, then ruptured, Dx?
A. Anal fissure
B. Hemorrhoid
C. anal abscess
D. fistula
Answer is: D

259. Alcoholic patient known to have cirrhosis presents with massive upper GI bleed.
They admitted him for EGD. What is an important medication to give before EGD?
A. Octreotide
Answer is: A

260. Post burn rehydration, which mirror good hydration?


A. Urine output 0.6ML/KG
B. CVP
Answer is: A, N.B: perfusion is mentoring by lactic acid except in burn by
urine output.

261. Patient RTA with head trauma, increased urine output, decrease in Urine
osmolarity increased blood osmolarity?
A. Central diabetes insipidus
Answer is: A

262. Pt came after accident Ex finding neck injury with tissue (coming out) Next step?
A. Oxygen mask
B. Mechanical ventilation
C. cricothyrotomy
Answer is: A

41
263. Colorectal surgeon performing a low anterior resection for CRC and the pelvis
won’t stop bleeding, so he consults a vascular surgeon and he did packing and it
doesn’t help what you will do?
A. Observe
B. Infrarenal clamp
C. Supraceliac clamp
D. Do the surgery and remove packing
Answer is: B

264. patient did appendectomy 6 years ago came with Symptoms of small bowl
obstruction X ray air fluids level Best diagnostic tool?
A. Us
B. CT
C. laparoscopic exploration
Answer is: B

265. female 50 y/o with leg non pitting edema with skin discoloration and thickening
ask about best test?
Answer is: Lymphoscintigraphy

266. a female patient came complaining of venous spider nevi, no other engorged
veins, she wanted to treated cosmetically, what investigation should be done
next?
A. CT venography
B. Vein plethysmography
C. Venous duplex
Answer is: C

267. female with LT leg swelling, intact distal pulse. Best investigation?
A. Doppler venous
B. Doppler arterial
C. CT angio
D. CT venum
Answer is: A

268. pt 3 hrs post op develop SOB?


Answer is: Atelectasis

269. Same scenario asks about mx?


Answer is: incentive spirometry

42
270. woman post lap cholecystectomy not sure for duration present with SOB,
(manifestation of atelectasis) and need mx?
A. bronchodilator
B. abx
C. secnti something
Answer is: if in first day its atelectasis > incentive spirometry, if came later
(5 days) mostly sub diaphragm abscess > drainage.

271. Question about pt admit for ventral hernia and with 7 days before admission do
retinal surgery which type of prophylaxis give for the prevent dvt?
A. pneumatic compression
B. pneumatic compression + enoxaparin
C. heparin
D. (I forget but i think is enoxaparin alone)
Answer is: A

272. Female, unilateral leg swelling, progressive, pitting edema >> Q. Investigation?
Answer is: Initial: Doppler, best: MRV.

273. Case of adult with open compound fracture?


Answer is: Urgent Debridement with External fixation

274. Pt with abdominal pain Mass in the adrenal 5 cm CT show hypodense and lipid
tissue normal hormonal labs what you will do?
A. reassure
B. Imaging guided biopsy
C. adrenalectomy
Answer is: C

275. management of Ventral hernia?


Answer is: sublly

276. epidural pedundal nerve block affect all except?


A. rectum
B. Anal
Answer is: A

277. Pt falls from 3m open injury, appropriate management?


A. Antibiotic
B. Debridement
C. Reduction
Answer is: A then B.

43
278. Pt with stones for cholecystectomy on US found to have asymptomatic 4.5cm
AAA, consult the vascular what they will say?
A. proceed with surgery and follow up with US
B. proceed with surgery and follow up with CT angio
C. Do CT angio prior surger
Answer is: A

279. Pediatric with supracondylar fracture, with no pulse?


A. Observation
B. K wire
C. Elevate hand
D. Surgical exploration
Answer is: D

280. years boy signs of testicular torsion, redness pain, absent cremasteric reflex,
previous episode same sx2 weeks (I think)?
A. Surgical exploration
B. NSAID
C. Antibiotic
D. Testicular elevation
Answer is: A

281. Pt with abdominal distension, vomiting on x-ray air fluid level?


A. Incarcerated
B. Strangulated
C. Obstructed
Answer is: C

282. Appendicitis collection 10x15?


A. open drainage
B. laparoscopic drainage
C. open appendectomy and drainage
D. laparoscopy appendectomy and drainage.
Answer is: percutaneous drainage + antibiotics.

283. Biopsy of the roof of maxillary sinus and came with loss of sensation of lower
eyelids and upper lip asking about which nerve affected?
A. infraorbital
B. infratrochlear
Answer is: A

284. the question of traveler with multiple falls?


Answer is: Subdural hematoma.

285. question on retrograde ureteroscope they found a dilated kidney, ureter,


bladder and proximal urethra?
Answer is: posterior urethral valve

44
286. Breast nodule with tethering skin and axillary LN involvement, BIRAD4 what to
do?
A. Needle core biopsy
B. Excisional biopsy
C. Needle aspiration
D. Mastectomy
Answer is: A

287. Breast mass BIRADS 3?


A. follow up
B. Core needle biopsy
C. mastectomy
Answer is: A

288. Post-surgery (for adhesion) patient presented with discharge from the wound,
what the appropriate management?
A. Regular dressing
B. Wound Exploration
C. US guided drainage
D. wound inspection
Answer is: D

289. 59 patient presents with presentation of cholangitis epigastric pain and fever. His
labs show: High ALP, direct bilirubin. CBD is 12 mm and there is perichole fluid.
What’s your management?
A. ERCP
B. MRCP
C. chosystectomy
Answer is: A

290. Dm, HTN male has Lateral malleolus ulcer what do you want to do next?
Answer is: check pulse

291. htn, dm pt with lateral malleolus ulcer with intact pulse, what is the diagnostic
tools?
A. venous us
B. CTA
Answer is: B, if there was ABI is more accurate.

292. Diabetic patient with pseudo hyper epithelialization in situ, what you should do?
A. Amputate toe
B. ulcer Debridement
C. follow up
D. Repeat biopsy
Answer is: D

45
293. The patient has critical limb ischemia, what is the golden standard investigation
to be done?
A. CT angiography
B. Conventional angiography
Answer is: B

294. Pt did hernia repair 5 years ago came with swelling at site of surgery, on
examination no redness no hotness only mild tenderness, -ve cogh implies?
A. Seroma
Answer is: bad recall (5 years ago?!), if post-op > A.

295. clear scenario of mastitis with no fluctuations what to give?


Answer is: Abx Dicloxacillin

296. Pt 60 y/o long standing DM and HTN has pain in buttock and weakness of muscle
On CT angio shows aortoiliac insufficiency. What is appropriate ttt?
A. vasodilator
B. Axilofemoral bypass
C. aortofemoral bypass
Answer is: C

297. mass in posterior triangle of neck?


A. Apparent thyroid
B. ectopic
C. metastatic
Answer is: C

298. Red nipple discharge mammography showed intraductal papilloma?


A. Core biopsy
B. Central duct excision
Answer is: A

299. female patient came complaining of swelling of the dorsum of the hand with
redness, she said she had knife prick prior to the swelling, what’s the diagnosis?
A. cellulitis
B. gas gangrene
C. necrotizing fasciitis
Answer is: A

300. patient had left chest pain and abdominal pain, he came after post splenectomy,
the pain increased with inspiration, on examination it was tender and chest had
reduced air entry to in the left side. What’s the diagnosis?
A. Lower lobe pneumonia
B. Subphrenic abscess
C. Super post splenectomy sepsis
Answer is: B

46
301. 25% blood loss what is mostly will be decreased?
A. Urine output
B. Pulse pressure
C. GCS
D. RR
Answer is: B, mostly A and B will decrease, but B decrease with some type
of shock like hemorrhagic as in this Q.

302. Patient with Crohn on ibuprofen and pentasa complaining of 3 perianal fistulas
that appeared a week ago, his abdominal symptoms are controlled with the
medications, everything is normal except for the 3 perianal fistulas on labs ESR 60,
what will you give the patient?
A. Azathioprine
B. Adalimumab
C. Budesonide
D. Infliximab
Answer is: D

303. Pt come to clinic, she is a typer and complain of left hand and fingers pain. It
shows defect in superficial palmer arch. Which artery is affected?
A. Ulnar artery
B. Radial artery
C. post inter
D. ant inter
Answer is: A

304. Case of perianal swelling, perrectal bleeding, investigated: it is 1 cm from anal


verge, biopsy: adenocarcinoma (rectal) Mx?
A. abdomen perineal resection
B. low anterior resection
C. C-chemo/radio
Answer is: A

305. leg numbness + absent of sm thing pulse or sensation?


Answer is: mostly is a case of compartment syndrome > fasciotomy

306. Patient presenting with Esophageal perforation post dilatation for achalasia, how
will you manage?
A. Surgical drainage and anastomosis
B. Esophagectomy
Answer is: Esophageal stent, if there high WBC or fever sign of peritonitis
> A.

47
307. Typical case of gall stone and came with sever RUQ pain after fatty meal for 6
hour and after 2 the pain gradual decrease, US multiple stone with normal
gallbladder wall what is the ttt?
A. lap chole
B. Ursodeoxycholic
C. Observation
Answer is: A

308. what is the mechanism of pain in compartment syndrome?


A. nerve hypoxia
B. muscle enlargement
Answer is: A

309. CXR showing plural effusion to the middle lobe, what would do?
A. Aspiration
B. Chest tube
Answer is: A, thoracentesis

310. man did inguinal hernial repair with mush, after few days, he has pain and
numbness at the mush side, no sign of inflammation at all? What is the
appropriate management?
A. Remove the mush
B. Nerve block
C. Give nonsteroidal
Answer is: C

311. When to do MRI to the brain in hyperprolactinemia?


A. Blurred vision
B. Double of serum prolactin
Answer is: A

312. Unclear Q about patient who came with nausea and vomiting and found out to
have appendicitis Q, what is the pathophysiology in this case?
A. Low cardiac index
B. Vasoconstriction
Answer is: B

313. A 60 years old patient complaining of dull aching swelling non tender found to
have a retro-peritoneal mass of a 20×20 cm size, and multiple masses in the liver.
Most likely diagnosis?
A. Liposarcoma
B. Germ cell tumour
C. Neural tube something
D. Lymphosarcoma
Answer is: A

48
314. Man with bronze skin, high ferrtin, other signs and symptoms, what test will
confirm the Dx?
Answer is: Liver biopsy

315. Pedia 4 days with jaundice and his brother had the same thing Direct bilirubin
and total bilirubin was high what is the diagnosis, High direct bilirubin in
neonates?
A. Biliary atresia
B. Choledochal cysts
C. Neonatal hepatitis
Answer is: A

316. 60 years old patient come with sudden onset of upper abdominal pain after a
few bouts of vomiting. Examination confirmed sick patient with tenderness in
epigastrium and supraclavicular subconscious emphysema. What’s Dx?
A. esophagitis
B. acute gastritis
C. perforated peptic ulcer
D. boerhaav’s syndrome
Answer is: D

317. Women has hepatitis C has a large mass (HCC) how to manage?
A. Resection
B. Chemotherapy
C. Radiation
Answer is: B

318. Patient is going to do laparoscopic cystectomy she is a smoker and taking OCP
for 6 years and suddenly developed tachycardia tachypnoea and hypotension,
what’s the Dx?
A. MI
B. PE
C. Septic shock
D. Hemorrhagic shock
Answer is: B

319. Male with colicky upper abdominal pain aggravated by morphine and its
derivatives, US shows mildly dilated CBD (I think it was 9 mm not sure) and dilated
intrahepatic ducts, what’s the Dx?
A. Sphincter of oddi dysfunction
B. Pancreatitis
Answer is: A

49
320. A patient underwent left lower parathyroidectomy for 1ry hyperparathyroidism
(adenoma). He presented 4 months later with depressed mood and fatigability.
Both PTH and calcium were high. What is the most common cause?
A. Parathyroid hyperplasia
B. Missed adenoma
C. New adenoma
D. Parathyroid cancer
Answer is: B

321. Pt with Complex regional pain syndrome has pain in wrist after carpal tunnel
surgery. Was seen before by a specialist and in pain clinic. What to do?
A. Counselling
B. physiotherapy
Answer is: B

322. which if following pt with primary hyperparathyroidism should undergo


parathyroidectomy?
A. >50 Y
B. Hx of osteoporosis
C. Very high pth8
Answer is: B

323. 35y old pt worried her mother had breast Ca and her sister had ovarian Ca. How
to screen her?
A. Mammo
B. US
C. BRCA
Answer is: A

324. Female in 20s has diabetic present with confusion. The patient has a long history
of type 1 diabetes. She also complains of periorbital swelling, rhinorrhea and black
necrotic spot over the face. labs show glucose 600 mg/ dl and ketones. Ct scan
shows obliteration of all the sinuses. Which of the following is the causative
organism? NEW20
A. Rhizopus oryzae
B. Candida albicans
C. Moraxella catarrhalis
D. Staph. Aureus
Answer is: A

325. elderly with lower gi bleeding, both upper and lower endoscopy r normal Next
investigation?
A. CT angio
B. capsule
C. MRI
Answer is: B

50
326. 29-year lady present with central abdominal pain from3 days and nausea,
vomiting 1 day ago. She did laparoscopic sleeve 6 years ago. Physical exam she is
dehydrated with distended abdomen and sounds?
A. incisional hernia
B. internal hernia
C. intussusception
D. adhesion
Answer is: D

327. DM Rt leg swelling .. Angiography showed diffuse disease 2 days later developed
firm tender partially mobile swelling irreducible nonexpansile below the inguinal
crease .dx ?
A. Psoas abscess
B. Saphena varix
C. Femoral hernia
D. Pseudoaneurysm
Answer is: C

328. Women with sudden loss of vision, Radiological finding found mass in pressure
over optic chiasm inside it hemorrhage when do surgery?
A. Urgent Resection now
B. Repeat Imaging after 1 week .
Answer is: Steroid then Urgent neurosurgical referral or A

329. Female came complains of headache + sudden painful loss of vision,


Examination: loss of vision in right eye. CT brain: pituitary haemorrhage MRI brain:
pituitary mass, hge + compressing on optic chiasma & cavernous sinus. Best next
step?
A. Close observation
B. Medical therapy
C. Inferior petrosal sinus sampling
D. Urgent neurosurgery referral
Answer is: D

330. Seizure with shoulder dislocation?


Answer is: posterior

331. Most common tumor in small bowl?


A. carcinoid
B. lymphoma
C. intestinal seroma
Answer is: A

51
332. Pathophysiology regarding patient with appendicitis complicated by
Appendicular mass (Abscess), patient was Feverish (38.7)?
A. Peripheral Vasoconstriction
B. Decreased Cardiac Index
C. Redistribution of Blood
D. Bradycardia
Answer is: A and C, fellow you heart.

333. Pituitary adenoma affecting vision?


Answer is: bitemporal hemianopia

334. Patient post hemorrhoidectomy came complaining of severe Supra pubic pain &
inability to pass urine, what is the most likely the cause?
A. inadequate analgesia
B. dehydration
C. urethral injury
D. hematoma
Answer is: A

335. Pt known case of emphysema, came with hematemesis they did endoscopy and
it was controlled, then pt is complaining of progress dyspnea. ABG: low ph, high
CO2 and high O2. What is your most appropriate management?
A. intubation and ventilation
B. give high oxygen
C. give sodium bicarbonate
D. Dexamethasone
Answer is: A

336. Post rectal surgery, patient is doing well, on Male complain of tingling sensation
over the dorsum of hand (thumb, index and ring fingers), also weak extension of
the fingers and wrist, what nerve is affected?
A. Radial
B. Ulnar
C. Median
Answer is: A

337. What is the least risk factor for gallbladder stones?


Answer is: Nulliparity

338. 50 years old come with sudden severe retrosternal pain radiating to back. Within
the minutes he come unconscious. He is a smoker and history of HTN. What’s Dx?
A. vasovagal attack
B. PE
C. tear of aortic intima
D. acute myocardial infarction
Answer is: C

52
339. Fracture, xray shows multiple saddle stones, recurrent. All labs are normal
except PTH was very high. What else you want to do? (Literally that was it)
A. Sistamibi scan
B. alkaline phosphatase
C. Intravenous pyelogram
D. If remembered
Answer is: A

340. History of hemorrhoidectomy surgery 3 weeks ago, presents with tachycardia


only. What’s the highest diagnostic value?
A. D dimer
B. CTA
Answer is: A, intermediate risk according to Wells craiteria.

341. Pregnant lady everything was normal but she complains of itching and had
elevated liver enzyme, what is the dx?
A. Viral hepatitis
B. Pregnancy cholestasis
Answer is: B

342. There was a case about elderly diabetic pt with 2-day history of abdominal pain,
bloody diarrhea imaging was done showed thickening of descending and sigmoid
colon with thumb appearance, what is the dx?
A. UC
B. Crohn disease
C. Ischemic colitis
D. Acute diverticulitis
Answer is: C

343. Case of abdominal pain and diarrhea colonoscopy showed non contagious
patches in the terminal ileum and colon Pathology shows transmural
inflammation what’s is the dx?
A. Ulcerative colitis
B. crohn disease
C. pseudomembranous colitis
Answer is: B

344. Patient had a hyperextension trauma, complains of distal phalanx pain and
tenderness in the volar aspect, he also feels tenderness in the palm, what's the
dx?
A. Rupture of flexor profundus
B. Rupture of flexor superficialis
C. intra-articular fracture of proximal phalanx
D. extra-articular fracture of distal phalanx
Answer is: A

53
345. smoker pt, c/o with dyspnoea and cough with hemoptysis, trachea wasn’t
shifted, x-ray showed massive right pleural effusion, what’s the cause?
A. Heart failure
B. T.B
C. Bronchogenic carcinoma with ipsilateral bronchus compression
Answer is: C

346. Young female suddenly developed jaundice and fatigue. She high ALP and high
bilirubin. No splenomegaly or hepatomegaly.US: no finding. MRCP: multiple foci of
stricture and dilatation. What is best initial next step?
A. Liver biopsy
B. Antinuclear antibody
C. Repeat US
D. Colonoscopy
Answer is: D

347. Case with numbness of anatomical snuff box cannot extended the
metacarpophalangeal with wrist drop radial nerve injury is suspected. Asking
about the site of injury of radial nerve?
A. Spiral grove of humerus
B. epicondyle
C. Axilla
D. Wrist
Answer is: A

348. Old man came to the ER with severe epigastric pain since 2h, he has arthritic
disease on meds?
A. Perforated peptic ulcer
B. Boerhaave syndrome
C. Gastritis
D. Esophagitis
Answer is: if there was peritonitis > A, otherwise > C.

349. Old patient has unilateral reducible hernia, there is no anything else?
A. Do laparoscope surgery
B. Open surgery
C. No need intervention
Answer is: C, note: because he is old

54
350. Pt presented with RUQ pain and jaundice, pt look unwell and abdominal tender
with no guarding, WBC high, high total bilirubin, high alkaline phos, US show
(multiple stones with no pericyclic fluid and dilated CBD with 1 cm, doesn’t
mention fever in scenario BUT show leucocytosis)?
A. ascending cholangitis
B. acute pancreatitis
C. acute cholecystitis
D. choledocholithiasis
Answer is: A

351. Female admitted for elective cholecystectomy, during the operation the surgeon
found big gastric mass. What to do?
A. Resect the mass
B. cholecystectomy only
C. end the procedure
D. cholecystectomy and mass resection
Answer is: B

352. 90-Year-old man with multiple comorbidities presented with many cancer in
stomach, lung and colon but the original site of the cancer is unknown. What’s
your most appropriate action?
A. resection of tumors
B. Palliative chemotherapy
C. Comfort care
Answer is: C

353. Pt develops hyperthermia generalized muscles rigidity, altered mental status,


tachycardia, HTN. Labs show elevated creatine kinase, he started metoclopramide
two weeks ago to treat diabetic gastroparesis. Which of the following drugs can
also cause these symptoms?
A. Fluphenazine
Answer is: A, N.B: Neuroleptic malignant syndrome case

354. pt admitted for Rt adrenalectomy because of Cushing what to give?


A. post op fludrocortisone
B. peri op corticosteroid
Answer is: B, N.B: Peri-op not Pre-op.

355. Most important risk factor for Breast ca?


A. AGE
B. Late menopause
C. Early menarche
D. Pregnancy after 30 yrs
Answer is: A

55
356. Female with lower limb edema. Distal pulses are intact. Only dilation of greater
saphenous vein tributaries. What is your investigation?
A. CT venography
B. Venous duplex ultrasound
Answer is: B

357. Patient with RUQ pain and tenderness and mass. Investigation showed
Entamoeba histolytica, and there’s abscess 12X14 cm I think, and ask about first
next step in ttt?
A. Metronidazole
B. Laparotomy
C. Percutaneous drain
Answer is: A.

358. Strongest indication of surgery in graves?


A. Pediatric
B. Presence of eye symptoms
C. presence of anti tsh
D. Failed antithyroid meds.
Answer is: B

359. When to do cholecystectomy in pregnant women?


A. 2nd trimester
B. 3rd trimester
C. After delivery
D. right now
Answer is: if cholecystitis or recurrent colic > D, otherwise conservative
treatment, best time for surgery in pregnancy > A.

360. Tubuvillous adenoma pink color most common site?


A. Rectum
Answer is: A, common in left side.

361. Patient came with history of Hashimoto’s disease then developed huge goitre.
Biopsy showed malignant thyroid cancer, what is the most common cause?
A. medullary
B. lymphoma
Answer is: B

362. Virchow’s lymph node Associated with which cancer?


A. Gastric cancer
Answer is: A

363. 32 YO I think female concerned about breast cancer, because her mother have it
and her sister has ovarian cancer, what to do?
A. BRCA
Answer is: Mammogram

56
364. Obese male tried lifestyle not effective He is diabetic and hypertensive what to
do, Weight 125, Hight 173, What next?
A. Medication lower weight
B. Bariatric surgery
Answer is: B, N.B: Because his BMI more than 35 and he is high risk
patient: diabetic and hypertensive patient.

365. HX of hernia 2 yrs back now present with symptoms of bowel obstruction, the
skin over the hernia is red in color what is the complication that the patient
develops?
A. Obstructed
B. Incarcerated
C. Strangulated
Answer is: C

366. Best modality to diagnose AAA?


A. US
Answer is: A

367. 25 years female with thyroid nodules TSH and T4 normal, FNA cytology done and
according to Bethesda classification (stage IV) what is the most accurate
management?
A. Lobectomy
B. hemithyroidectomy
Answer is: A

368. What is the favorable place for AV fistula?


A. brachial artery basilic vein
B. brachial artery cephalic vein
C. radial artery basilic vein
D. radial artery cephalic vein
Answer is: D

369. GIST tumor in the Body of the stomach, it’s size about 3cm what you will do?
A. total Gastrectomy
B. WLE with clear margin
C. Observation
Answer is: B

370. pt with non-insulin dependent DM Come with upper back swelling lesion +
multiple opening discharge (pus I think) What is the diagnosis?
A. Abscess
B. Carbuncle
C. Furuncle
Answer is: B

57
371. Pt with hip fracture, what prophylactic should be given?
A. Enoxaparin sc bid
B. heparin sulphate 10.00 BID
Answer is: A, the correct answer is 40 OD.

372. Tibial shaft fracture after RTA High velocity And absent dorsalis pedis pulse and
posterior tibial pulse, numbness in the lower limb In severe pain What is the
appropriate management?
A. Splint and elevation
B. Fasciotomy
Answer is: B

373. History of HTN, heart failure and admitted for hysterectomy and received normal
saline because poor oral intake, 4 days later nurse noticed decrease in saturation
and SOB On examination: crackles. How to prevent this complication?
A. Daily fluid assessment
Answer is: A

374. patient came after limb trauma, with severe pain and paranesthesia between his
toes and it was pale X ray shows fractures, intra-compartmental pressure was
35mmhg what to do?
A. internal fixation
B. closed reduction
C. external fixation
D. external fixation with 5 fasciotomies
Answer is: D

375. Most important thing to do before reduction of the fracture?


A. Check the pulse
Answer is: A

376. Female 30s asking for cosmetic treatment of varicosity in her thigh,
asymptomatic, no other varicosity, how would u investigate?
A. Duplex
B. CT venography
C. No need for further investigation
Answer is: A

377. Patient has angel of mouth pain for 2 days , he had a history of prior
laparoscopic cholecystectomy before 8 days, On vital the patient was alert and
stable except for mild fever, what is the most appropriate next step?
A. antibiotics
B. CT
C. x-ray
D. paracetamol
Answer is: A, case of post op parotitis.

58
378. Thenar atrophy caused by which never?
A. median
B. Ulnar
C. Radial
Answer is: A

379. old male with abdominal pain and was not passing gas he presented to the ER 24
hrs ago. WBC 20,000. Imagine showed Y sign?
A. observation
B. colonoscopy
C. sigmoid resection with colostomy
Answer is: C, pt with sign of peritonitis need surgery.

380. Breast feeding mom complains of unilateral inverted nipple when she try to
adjust the nipple it slit horizontally, what’s the dg?
A. Duct Ectasia
B. Malignancy
Answer is: A

381. Known case of hepatitis C complain of RUQ pain, aFP was elevated Ct show right
liver lobe mass or something with hyper vascular?
A. Hemangioma
B. HCC
C. Cholangiocarcinoma
D. Adenoma
Answer is: B

382. Q- pain, paranesthesia, pallor, history of MI before weeks, diagnosis?


A. acute arterial thrombosis
B. acute arterial embolism
C. DVT
Answer is: B, case of Acute limb ischemia

383. Q: Patient have trauma in right chest and have pneumothorax and do chest tube
after 15 min water seal bottle is filed with 2 litter of blood, BB: 90/60, HR:100,
Right :22 come what’s next step?
A. Thoracotomy
B. Thoracostomy tube
C. CT abdomen
Answer is: A

384. After trauma Best test to rule out cervical spine?


A. MRI cervical
B. AP x ray of cervical
C. CT
D. clinical exam
Answer is: C

59
385. Patient did retinal operation DVT prophylaxis what to give?
A. heparin
B. enoxaparin
C. mechanical compression
Answer is: C

386. Adult case of intussusception, asking about management?


A. barium enema
B. surgical resection
C. surgical reduction
Answer is: B

387. GIST tumor in 25 yr man it was 4X3 CM what r u going to do?


A. (wedge) excision with safe margin + imantib
Answer is: A

388. Case of pain on defecation, patient was pale on ex. and the doctor was trying to
do PR examination and was not able to because of pain?
A. examine under anesthesia
B. LIS
C. Internal LIS
Answer is: A

389. Long case about thyroid mass with laps and the bottom-line question was. What
is the best Tx for the palpitation?
A. BB
B. PTU
C. Methazole
Answer is: A

390. Pt with liquid dysphagia and food reguarge What is the test with highest
diagnostic yield?
A. barium
B. CT
C. endoscopy
D. lower esophageal sphincter manometry
Answer is: D

391. Post appendectomy female came with LR abdomen mild tenderness Ex Normal
by CT there is 2*2 collection in Retrocecal?
A. Exploring laparotomy
B. percutaneous drainage
C. laparoscopic
D. conservative with Antibiotic
Answer is: D

60
392. Case of internal hemorrhoid grade 4, what is the most appropriate treatment?
A. open hemorrhoidectomy
B. rubber band ligation
Answer is: A

393. Adult pt feels groin pain especially with exercise. Exam of groin and genitals
normal, next step?
A. Assess after 3 months
B. Us
C. Ct
D. MRI
Answer is: D

394. A 62 YO man medically free c/o 1 day hx of 4 episodes of hematochezia no abd


pain, constipation and wt loss Dx?
A. Diverticulosis
B. Cancer
C. Ischemic colitis
D. Internal piles
Answer is: A

395. Case of melanoma, male with skin ulcer severe pigmentation indurated what is
the treatment
A. biopsy
B. excision biopsy
Answer is: B

396. Trauma patient with a wound on his thigh, subcutaneous fat is lost and
vasculature underneath is exposed what provides the best management?
A. Debridement with primary closure
B. Primary repair
C. Debridement with secondary graft
D. Debridement with vacuum assisted closure
Answer is: C

397. adrenal adenoma discovered accidentally in CT. Most common adrenal mass
discovered accidentally?
A. sub clinical Cushing
B. conn's
C. cancer
D. non-functional adenoma
Answer is: D

61
398. 1 cm stricture in ileocecal valve what do u do?
A. medical management
B. right colectomy with stoma
C. stricturoplasty
D. exacion with ileostomy
Answer is: D

399. old with 2nd degree hemorrhoids pale and fatigue no signs of cancer,
hemoglobin was low, what is the cause of anemia?
A. Hemorrhoids
B. Rectal cancer
C. Cecal cancer
D. Sigmoid cancer
Answer is: Asymptomatic anemia without lower GI bleed > C, if with lower
GI bleed (streak of blood) > B.

400. Hashimoto thyroiditis increase risk of?


A. papillary thyroid cancer
B. thyroid lymphoma
Answer is: B

401. Patient had Afib came with 4 hours of pain, pale , cold limb swelling , absence
pulse (Sx of acute limb embolism) What’s the most appropriate investigation?
A. CT angiogram
B. Conventional angiography
C. embolectomy without imaging
Answer is: the correct answer is D which is arterial duplex (in other recall).

402. Pt with s&s of intestinal obstruction imaging shows proximal ilium diltation and 3
strictures involve (30 cm) from proximal ilium. No constitutional symptoms
mentioned. What’s the dx?
A. TB enteritis
B. Crohn’s
C. Small intestine lymphoma
D. GIST
Answer is: B

403. Clear case of hyperthyroidism with exophthalmos and on anti-thyroid drugs she
doesn’t improve. Scan shows multinodular goiter, what’s your management?
A. near total Thyroidectomy
B. hemithyroidectomy
C. iodine
Answer is: A

62
404. 9 years old presented with abdominal distention and pain, he was found to have
intussusception, wat is the Mx?
A. Surgical resection
B. Surgical decompression
C. Colonoscopy
D. Enema
Answer is: D, in adult age > A.

405. DM and HTN known case patient, presents with severe epigastric pain radiating
to back for 6 hours, CXR shows left air under diaphragm, most likely diagnosis?
A. Perforated peptic ulcer
Answer is: A

406. Pt heavy smoker with symptoms of GERD, upper endoscopic examination show’s
squamous cell with high-grade dysplasia, next step in management?
A. Council smoking cessation
B. Endoscopic mucosal resection
Answer is: B

407. Pt w/t mitral valve prolapse and she will undergo for thyroidectomy?
A. Give amoxicillin to prevent infective endocarditis
B. No need for prophylaxis
Answer is: B

408. hypotension, raised JVP, muffled heart sounds?


A. Cardiac tamponade
Answer is: A

409. Long scenario about pt had dialysis 3 day ago central line and now came again
for dialysis peripheral line nurse mentioned he had fever and infection in site of
previous dialysis site what to do?
A. culture and iv abx and stop dialysis
B. culture iv abx and remove the center line
Answer is: B

410. Rupture duodenal ulcer did graham patch after 8 days: fever Lung: Dec breathing
in RL, Central line and peripheral line culture +ve diplocci Abdomen: soft lax Next
step?
A. Us guided aspiration
B. Remove central line
C. Laparotomy
D. Laparoscopy
Answer is: B

63
411. Double vision, when he looks straight the lt eye deviated to nose side, Lt. eye
cannot look laterally?
A. Rt. 6 CN palsy.
B. Rt. 3 CN palsy.
C. Lt. 6 CN palsy
D. Lt. 3 CN palsy
Answer is: C

412. Male patient with recurrent hematemesis, no other complain, Vitaly stable, labs
show anaemia, LFT normal. (not mention any other symptoms) What’s your
diagnosis?
A. esophageal varices
B. peptic ulcer disease
C. erosive gastritis
D. Mallory weiss tear
Answer is: C

413. What is the best mesh used in Ventral hernia repair?


A. Sublay mesh
Answer is: A

414. Female post gastric bypass surgery (i think 8 weeks) complaining of recurrent
vomiting and bad mood, all labs normal, CT scan normal, what’s your next step:
A. exploratory laparotomy
B. reassure
C. psychiatric consultation
D. behavioral and psychological treatment
Answer is: B

415. During Lap cholecystectomy you found AAA 4.5 cm what you will do?
A. Do the surgery and FU with US
B. Do the surgery and follow up with CTA
C. Do CTA before the surgery
Answer is: A

416. Case of secondary pneumothorax due to COPD 2 cm, what you will do?
A. Observe
B. O2 mask
C. Chest tube (If symptomatic)
Answer is: C

64
417. Pediatric patient as a case of trauma Presented to ER complaining of decrease air
entry in hemithorax Tympanic percussion in hemithorax, BP low, Tachycardia
What is your management?
A. Observe
B. Discharge
C. Needle decompression in the affected side
D. Chest tube in the affected aside
Answer is: C

418. Appendicular abscess, treated conservatively and the patient improve, what you
will do for him?
A. lap appendectomy after 12 weeks
B. Open appendectomy after 12 weeks
C. Colonoscopy after 6 weeks
Answer is: if old age > C, otherwise > A.

419. Case about female with history of gallstone pancreatitis 6 weeks back, present
with epigastric pain and fever Ct report: Collection “large” 14x16 (something like
this). High WBC what is the management?
A. Percutaneous aspiration
B. Endoscopic aspiration
C. Open aspiration
Answer is: A

420. Case about Gun shot in the chest, the patient was managed with tube
thoracostomy for pneumothorax,15min late the there was 2 litter of blood from
the thoracostomy. What is the management?
A. Thoracotomy
B. Tube thoracostomy
C. Angioembolization
Answer is: A

421. Young 24 man in military, with bilateral inguinal hernia, management?


A. Lap with mesh
B. open with mesh
C. open without mesh
D. observe
Answer is: A

422. Most common site of thoracic aortic blunt trauma?


A. Aortic arch
B. aorto-ligament
C. proximal to subclavian artery
D. distal to left subclavian artery
Answer is: D

65
423. Patient with DM and HTN presented with ingrown toenail, what is the thing you
must do before the operation?
A. Check peripheral pulses
B. Check the other toenails
C. Take swab culture
D. Antibiotics
Answer is: A

424. biliary colic is associated with?


A. amylase
B. Bilirubin
C. Cholecystokinin
D. don’t remember the last one
Answer is: C

425. 61-year-old came for elective hernia repair, found to have raised jvp and lung
crackles or infiltrates how to manage?
A. do open repair as scheduled
B. do laparoscopic repair as scheduled
C. optimize pt sx then reassess
D. never do surgery unless incarcerate
Answer is: C

426. Pt after RTA presented with mandible and maxillofacial fracture how to keep air
way breathing more save??
A. tracheostomy
B. nasal o2
C. Cricothyrotomy
Answer is: C

427. Known drug abuser found in deep coma and cyanosed, vitally unstable. Initial
management?
A. Hx from the family
B. Mechanical ventilation
C. CT brain
Answer is: B

428. Battle's sign?


A. Basal skull fracture
Answer is: A

429. Trauma patient has basal skull fracture that includes the jugular foramen what
will the patient have?
A. ipsilateral vocal cord paralysis
B. Paralysis of the muscles of mastication
C. Can’t abduct eye
Answer is: A

66
430. Pancreatitis 6 weeks ago. Now she has epigastric tenderness and cannot tolerate
food with vomiting each time. By ultrasound you found large about 12X10 mass
With thick wall and fluid inside. Labs: 346 amylase, WBC 15k. What is the
diagnosis?
A. Pseudocyst
B. Abscess
C. Walled off necrosis
Answer is: B, if homogeneous in US > B, if heterogenous on US > C.

431. Patient bleeding from angiodysplasia left side, Mx?


A. Left hemicolectomy
B. Angio-embolization
C. Endoscopic laser ablation
Answer is: stable > C, unstable > B.

432. Bell’s palsy nerve affected?


A. 7th
Answer is: A

433. Pt with history of 9-month dysphagia to solid and liquid 4 Episode hematemesis
lower sternal chest pan What is your diagnosis?
A. Esophageal varices
B. Achalasia
C. SCC of esophagus
D. PUD
Answer is: C

434. Splenectomy what to expect post op to be low?


A. insulin
B. glucagon
Answer is: A

435. 19 year old pt , known case of cystic fibrosis, presented with abdominal pain,
weight loss, malnourished, pale stool?
A. acute pancreatitis
B. chronic pancreatitis
C. pancreatic cancer
Answer is: B

436. Deep wound in anterior thigh 10 cm, bleeding, how to control bleeding?
A. direct pressure on the wound
B. apply tourniquet
C. pressure on the femoral vessel above the wound
D. referee to vascular surgeon
Answer is: A

67
437. 30 yo multigravida mother complain of perianal pain and absolute constipation
for 2days. During exam anal cushions are irreducible and inflamed. Vitals normal
BP, tachycardia, normal Temp. What is the diagnosis?
A. perianal abscess
B. Thrombosed pile
C. Anal cancer
Answer is: B

438. male, presented to ER after motor vehicle accident, patient was conscious and
alert, normal heart sounds, clear chest, and warm extremities BP 90/- HR 70 bpm
(no other information). What type of shock?
A. Hypovolemic
B. Septic
C. Cardiogenic
D. Neurogenic
Answer is: D

439. After sleeve gastrectomy 19 y pts has jaundice and pain, N&V, sluggish biliary
Manegent?
A. Lap chole
Answer is: A

440. Newborn has respiratory distress and x ray show shifting of mediastinum in
other side and increase darkness?
A. pneumothorax
Answer is: A

441. Pleural effusion due to Malignancy did multiple pleural tap what will you do
now?
A. Chemical pleurodesis
Answer is: A

442. Male patient presented with acute limb pain, known history of a fib. On
examinations absent dorsalis pedis pulse with intact femoral. Which of the
following is the most appropriate next step?
A. Amputation
B. Catheter directed thrombolysis
C. Give heparin and observe
Answer is: pain only > C, plus sensory symptom > B, plus motor symptom >
embolectomy.

68
443. Similar scenario like the above Q, which of the following most important
investigation?
A. CT angio
B. Conventional angio
C. Vascular duplex
D. Embolectomy without imaging
Answer is: C

444. Polytrauma patient was sent to ER for stabilization, which of the following is
most important to stabilize the patient?
A. Establish Airway
B. Put cervical collar
C. IV access
Answer is: A

445. 27 females presented with acute abdominal pain. She was requested for CT.
Which of the following is the most important?
A. Full bladder
B. Pregnancy test
C. Previous CT scans
D. Volume status
Answer is: B

446. Patient was going for weight reduction surgery. On endoscopy found grade III
hiatal hernia: What is the procedure of choice?
A. Sleeve gastrectomy
B. Roux-en-Y gastric bypass
C. Ballon dilation
Answer is: B

447. Pt breastfeeding her child develop erythema and swelling of the right upper
quadrant of breast .PE tenderness and no LN involvement, what is the
management?
A. flucloxacillin
Answer is: A

448. Elderly pt with chronic HCV infection present with abdominal pain and jaundice.
Labs show high alpha feto protein What is the diagnosis?
A. HCC
B. Peptic ulcer
Answer is: A

69
449. Pt had cholecystectomy and cystic duct was injured how would you repair it?
A. Hepato jejunostomy
B. Hepato deudeontomy
C. Choldejejuonestomy
D. Cholo deudentomy
Answer is: A

450. MI patient just had surgery how to treat?


A. Primary angioplasty
B. Thrombolytics
Answer is: A

451. Anal fissures with skin tag not respond to drug, want procedure?
A. Internal sphernctomy
B. External sphrenctomy
C. Anal curettage with remove tag
Answer is: A, Lateral internal.

452. BMI: Hight 175 wight 125?


Answer is: bariatric surgery

453. Regional complex pain syndrome after carpal tunnel surgery release, complain of
wrist and hand pain, been treated with ortho, and pain clinic, what is the
appropriate mx?
A. Opiate
B. Triptan
C. Physiotherapy
Answer is: C

454. Case of pancreatitis pseudo cyst ask about management of it?


Answer is:
▫ Depend if it is infected > percutaneous drainage
▫ Not infected > internal drainage by endoscopic drainage

455. Pt with hip fracture, what prophylactic should be given?


A. Enoxaparin sc bid
B. heparin sulphate 10.00 BID
Answer is: the correct answer is Enoxaparin 40 once daily

456. Colorectal surgeon performing a low anterior resection for CRC and the pelvis
won’t stop bleeding, so he consults a vascular surgeon and he does?
A. Heavy packing of pelvis
B. Arteriography intra operatively
C. Infraceliac clamp
Answer is: A, if he did pack and vascular is arrive do B.

70
457. A patient post total thyroidectomy developed neck swelling 5 hours after the
surgery most appropriate management?
A. tracheostomy
B. bedside evacuation
C. percutaneous aspiration
D. observation
Answer is: B

458. Patient involved in RTA was transmitted to nearby limited facilities hospital and
the physician decided to refer him to advanced facilities hospital Patient is
unconscious: What is the most important thing he should be maintain?
A. intubation
Answer is: A

459. Pt did sleeve and comes after 4 wks severe abd. Pain?
A. Exploratory
B. Imaging
Answer is:
▫ Depends on the Case Stable > CT
▫ Unstable > Surgery

460. Neck mass biopsy showed normal follicular thyroid cells?


A. Reassure
B. Surgical refer
Answer is: B

461. Post-operative of appendicitis 8 days ago, came in clinic with seroma?


A. Regular wound dressings
B. Aspiration
C. CT
Answer is: A

462. Which disease benefit from splenectomy?


A. B thalassemia
B. A thalassemia
C. Sickle cell
Answer is: hereditary spherocytosis.

463. post thyroidectomy complication “low pitch voice” what is the cause, Injury to
which nerve?
A. Superior laryngeal nerve
Answer is: A

464. Case of “anal tear “chronic constipation ...etc ask about Dx?
A. Anal fissure
Answer is: A

71
465. year-old female presents with breast mass, ill defined. What is you next step?
A. Mammogram
Answer is: A

466. Burn case black soot over nostrils and mouth 40% carboxyhemoglobin. (Carbon
monoxide toxicity)?
A. Intubation and ventilation.
Answer is: A

467. Long case of breast cancer then in pathology: BIRADS IV what to do next?
A. Core needle biopsy
Answer is: A

468. A pregnant with unilateral breast lump what to do?


A. Bilateral US
Answer is: A

469. Trauma patient has injury In membranous urethra, initial Mx?


A. Suprapubic catheter
Answer is: A

470. 50+ man with a picture of intestinal obstruction, X ray shows: hugely dilated
bowel pointing to left upper quadrant and collapsed colon?
A. Sigmoid Volvulus
B. Cecal Volvulus
Answer is: B

471. HX OF hernia 2 yrs back now present with symptoms of bowel obstruction the
skin over the hernia is red in color what is the complication that the patient
develops?
A. obstructed
B. incarcerated
C. strangulated
Answer is: C

472. Case of perianal swelling, per rectal bleeding, on examination: there is 1 cm mass
from anal verge, biopsy shows adenocarcinoma, mx?
A. Abdominal perianal resection
B. Low anterior resection
C. Chemo/radio
Answer is: A

473. Old patient in ICU with acalculous cholecystitis, best management?


A. Cholecystomy tube
B. Lap chole
C. Open chole
Answer is: A

72
474. Burn patient presented RUQ pain, dilated gallbladder, thick wall and pericyclic
fluid, what is the management?
A. Cholecystectomy
B. Cholecystostomy tube
C. MRCP
Answer is: B

475. male involved in RTA in ER he is fully conscious and no abnormality in vital signs
admitted for observation and a FAST scan done, what do you expect to find?
A. pneumothorax
B. major vessel bleeding in the thorax
C. Intra peritoneal fluid
Answer is: C

476. pt underwent pneumodilation for achalasia cardi, present after a while with
signs of oesophageal perforation ask about mx?
A. Esophageal stent
B. esophagectomy
C. Surgical drainage + anastomosis
Answer is: stable > A, if unstable or high temp or WBC > C.

477. Patient after Percutaneous transhepatic cholangiography (PTC) developed upper


GI bleeding what is most appropriate investigation to do?
A. CT
B. US
C. MRI
D. Angiography
Answer is: D

478. Biopsy report showed Malignant phyllodes tumor what is most appropriate
next?
A. CT chest with contrast
B. Chemotherapy
Answer is: hormone receptor status, if not there go with A.

479. TSH was very very low and T4 was 21 (high) in US report they revealed two solid
thyroid nodule and ask about the most appropriate next step management?
A. Thyroid scan
B. Total thyroidectomy
C. FNA from both of them
D. FNA from the larger one
Answer is: A

73
480. pt presented with abdominal pain n exam there was a pulsatile mass and
tenderness with deep palpation what is most appropriate diagnostic test?
A. US abdomen
B. CTA
C. MRA
D. Conventional angiography
Answer is: A

481. Trauma patient, with description of internal bleeding, what type of shock?
A. Hypovolemic
Answer is: A

482. A case of a 40 y/o man, w/Charcot knee & he was informed about the need for
above the knee amputation & he refused. What is the appropriate thing to do?
A. Evaluate his mental capacity to decide
B. Refer him to another hospital
C. Ask him to sign the discharge form against medical advice
D. Consult the hospital ethical committee
Answer is: C

483. Case of breast lesion, w/ BIRADS III (Mostly benign). What’s the appropriate
thing to do?
Answer is: fellow up 6 months

484. 10 days post-partum breastfeeding lady, c/o fever, chills, & swollen left breast.
O/E her left breast was ingorged, cracked nipples & there was fluid-containing
space. What’s the most likely dx?
A. Breast abscess
B. fibroadenoma
Answer is: A

485. A 20 y/o lady came to the clinic c/o bilateral breast mass, it was mobile, oval in
shape, measuring about 2x2, well-defined borders. What’s the most likely dx?
A. Fibroadenoma
Answer is: A

486. subphrenic abbess treatment (he said collection under the diaphragm after
cholecystectomy and ask for tx)?
Answer is: drainage

74
487. Hx of sleeve bypass present with sx of leak Severe pain and Mild hypotension
Next step?
A. Endoscopy
B. Us
C. Laparotomy
D. Dx laparoscopy
Answer is: C

488. Patient 3 day after sleeve complaining of pain in left upper quadrant on
examination there is mild tenderness in left upper quadrant all his vitals were
normal What is the cause of his presentation?
A. gastric leak
B. hemorrhage
C. inadequate analgesic
D. infection
Answer is: C

489. Pt with thyroid nodule (that’s it no other information given, no TSH) asking what
is your next step?
A. FNA
B. Thyroid scan
Answer is: B, if there was symptom the answer will be according to hyper
or hypo symptom.

490. Pt with thyroid nodule and low TSH / high T4? Next step
A. FNA
B. Thyroid CT scan
C. Thyroid iodine Scintigraphy
Answer is: C

491. Mastitis most common organism?


A. Staph aureus
B. Streptococcus
Answer is: A

492. Pt had repeated vomiting then developed chest pain and supraclavicular
emphysema dx?
A. Boerhave syndrome
Answer is A

493. Gunshot lateral to nipple and Through the scapula pt became hypotensive lung
clear quite heart sound distended neck veins what to do?
A. Pericardiocentesis
B. Thoracotomy
Answer is: A, if there was IV fluid is first then A.

75
494. Pt i forget his age accidently on US we found gallbladder polyp 0.6 cm no
gallstone What to do?
A. consult surgery
B. reassurance
C. us f/u after 6 wk
Answer is: C

495. Pt with ulcer on lateral malleolus has HTN but Not diabetic what to do?
A. Venous duplex
B. Arterial duplex
C. Check pulse
D. ABI
Answer is: C

496. Patient had MVA and came with Barret sing, indicates what?
A. fracture base skull
Answer is: A

497. Patient had fractures in leg with severe pain, parathesis, and decreased pulses.
What will you do?
A. fixation of the fracture
B. fasciotomies
Answer is: B

498. Patient has anal fissures and polyp. What will you do?
A. cryotherapy
B. lateral external sphincterotomy
C. lateral internal sphincterotomy
Answer is: C

499. 30 something left none pitting oedema in end saying no history of trauma injury
but there is thickening if skin and dark color what u order investigation?
A. ct angio
B. venous duplex
C. arterial duplex
D. us
Answer is: B

500. Stab wound in the neck after they managed her she suddenly complained she
can’t breathe on examination you find swilling in the neck what’s the next step
A. Open wound in the ward
B. Do ct chest and abdomen
C. Give epinephrine
D. Give bronco dilates
Answer is: A

76
501. Direct question was female patient with intraductal papilloma and asking color
breast discharge?
A. white
B. green
C. yellow
D. red
Answer is: D

502. young Patient With history of use NSAIDS long time for joints pain now this
patient comes complaining of severe abdominal pains with sines of shock and
they gave you X-ray pic ask you about the diagnosis?
Answer is: duodenum perforation

503. Patient with chronic peripheral artery symptoms What's best next to do before
moving him to ER?
A. check pulse
B. check another toe
C. C and D were investigations
Answer is: A, if there was absent pulse so check other toe.

504. Man with left chest contusion, can shrug shoulders but not move elbow or lower
left leg, hypotension. What is the cause of hypotension?
A. Cardiac tamponade
B. Tension pneumothorax
C. High spinal cord injury
Answer is: C

505. Pt post cholecystectomy, had jaundice, RUQ discomfort, high bilirubin, high ALP,
u/s showed 2×3 subhepatic collection, CBD 9mm. What's the cause?
A. CBD injury
B. retained CBD stone
C. slipped clamp (something like that)
Answer is: B

506. Pt with penetrating trauma in the right anterior axillary line just below the right
costal margin, next step?
A. exploration
B. CT
Answer is: according to other answer, and patient is stable or not. Stable >
FAST if not there CT, if unstable > A.

507. Pt with penetrating trauma and wound located anterior to the left ear near the
angle of mandible, next step?
A. exploration
B. CT
Answer is: B

77
508. ultrasound reveal sludge in gallbladder and pt has symptoms, next step?
A. lab chole
B. ERCP
Answer is: A

509. After a successful cleps of dudenal ulcer who long are going to give iv ppi for a
medically free pt?
A. 24
B. 36
C. 48
D. 72
Answer is: D.

510. Management of grade IV hemorrhoid?


A. Hemorrhoidectomy
Answer is: A

511. 32 females came with breast mass, in examination mass is small, mobile, rubbery
what is most DX?
A. Fibroadenoma
Answer is: A

512. Male came severe abdominal pain and distension after 4 weeks of sleeve
gastrectomy what is the best investigation to order?
A. CT abdomen
B. US
C. abdomen X-ray
Answer is: A

513. Years old female came with breast mass mammogram show BIRAD III what is the
next step?
A. follow up in 6 months
Answer is: A

514. Male with previous history of chronic pancreatitis after 4 week came with
epigastric pain, vital stable no fever, CT show pseudocyst what is the next step?
A. observation
B. Internal drainage
C. Percutaneous drainage
Answer is: B

515. Perianal low fistula with close opening management?


A. Fistulogram
B. Fistulotomy
C. MRI
Answer is: B

78
516. What lab investigation would to order in pt with pulsatilla abdominal mass?
A. Amylase
Answer is: A

517. patient with Hep C comes with HCC single mass 6x6 What is the management?
A. Chemotherapy
B. Radiotherapy
C. Resection
D. Palliative therapy
Answer is: A

518. The same question of the crohns patient with the stricture that is 1 cm away
from the ileocecal valve? What is the management?
A. Steicturoplasty
B. Ileostomy
C. Medical management
D. ileocecal resection with ileostomy
Answer is: D

519. Appendicitis scenario: what should you do next?


A. Appendectomy
B. US
C. CT
Answer is: in adult > C, in child and pregnant > B, in high Alvarado score >
A.

520. pt with picture of sever pancreatitis in ICU what is the we will give?
A. vit c
B. vit K
C. the rest of Options about coagulation factor
Answer is: B

521. pt with bypass Roxn y after that feel pain in the shoulder fever and Ringer what
to do next?
A. endoscope
B. Abdomen US
C. diagnostic laparoscopic
D. CT with contrast
Answer is: D

522. old pt with jaundice, dark urine, pale stool and palpable gall bladder what is
diagnosis?
A. Ktalskin tumour
B. periampullary tumour
C. CBD stone
Answer is: if there was pain > C, if painless > B.

79
523. pt with picture of biliary pancreatitis after 3days Improve what to do?
A. cholecystectomy after 6-8 weeks
B. ERCP
C. cholecystectomy now
Answer is: C

524. 2 * 3 pus discharge wound what to do?


A. wound department
B. IV Abx
C. Observation
Answer is: B

525. Non tender mass non pulstile with high lipid 250?
A. Pudocyst
Answer is: A

526. pt with weakness and loss of reflex in arm, I am forget the senior but I remember
the options so you should read about options?
A. wplish injury
B. rotator cuff tendinitis
C. polymyalgia rheumatica
D. prolapse
Answer is: D, cervical prolapse.

527. orthopedic case develops dyspnea and patiche in neck Dx?


A. fat embolism
B. pulmonary embolism
Answer is: A

528. orthopedic case with gas gangrene what is the cause?


A. Clostridium perfringens
Answer is:

529. lactating mother with breast redness, hotness and mass 5cm mx?
A. I & D
B. aspiration
Answer is: if there thinning of skin > A, if no > B.

530. 4-month infant with bilateral inguinal hernia next?


A. Herniotomy
B. Lap repair with mesh
C. Open repair with mesh
Answer is: A

80
531. 55 patients came to ER with abdominal distention and vomiting previous history
of ventral hernia repair with mesh U/S Target sign and stricture in terminal ileum?
A. Adhesion
B. late onset chrons
C. mesntric ischemia
D. strangulated ventral hernia
Answer is: B

532. 88-year patient came to ER with sudden onset of epigastric pain radiate to the
back with history of cholethliasis on examination generalized abdominal
tenderness and guarding what to do next?
A. Erect x Ray
B. CT abdomen
C. Ultrasound
D. barium study
Answer is: A

533. Blunt trauma to the urethra what to do?


A. Foly catheter
B. suprapubic catheter
C. i don’t remember
Answer is: B

534. trauma to neck with devitalized lesion?


A. Thoracotomy
B. Cricothyroidotomy
C. Intubation
D. chest tube
Answer is: C

535. case of tension pneumothorax what to do?


A. Chest tube
Answer is: Needle decompression, if not there > A.

536. What indicates adequate Resuscitation?


A. Central venous pressure
Answer is: Mix venous oxygen saturation, if not there A

537. patient with trauma to face with fracture mandible resuscitation with?
A. laryngeal mask
B. Cricothyroidotomy
C. Oropharyngeal tube
D. Nasopharyngeal tube
Answer is: B

81
538. alcoholic with RUQ discomfort and mass in hypochondrium. Imaging was done
and report shows 6x6cm liver mass. What do you do?
A. Chemotherapy
B. Radiotherapy
C. Resection
D. thermal ablation
Answer is: A

539. Case of flial chest with confusion and all labs are normal except o2 sat is 88%,
what is the management?
A. Fluids
B. elective intubation
C. chest tube
D. belbectomy
Answer is: B

540. Man was reduced from burning house, he was mildly confused and his hair and
nose were signed?
A. admit to ICU and observe
B. ignore, medici legal issues
C. oxygen mask
Answer is: according to D (intubation), but A is correct.

541. Pt fall 50 feet presented to er with decrease air entry to left side Xray show right
hip fracture What is the next step (Pic of chest x-ray)?
A. Tube thoracostomy
B. Thoracotomy
C. Iv fluid
Answer is: A, according to X-ray (could be pneumothorax and answer >
needle decompression, if hemothorax > A).

542. Hip fracture in elderly, what to give?


A. Heparin
B. Enoxaparin
Answer is: B

543. Scenario describes a patient with abdominal aortic aneurysm what will you ask
for?
A. Cl
B. ABG
C. Amylase
Answer is: C

82
544. Elderly with bleeding per rectum, have anemia, otherwise normal Rectal exam:
1cm mass above anal verge CT scan: 3cm mass within rectal wall with LN
enlargement Biopsy: adenocarcinoma What’s your management? NEW20
A. Chemotherapy
B. Low anterior resection
C. Abdominal perineal dissection
Answer is: C, best is neoadjuvant chemo then surgery, if negative LN > B.

545. 20-year-old post appendectomy day 8, presents with purulent discharge from
the wound site, he is febrile, abdomen soft with tenderness over the wound +
redness, PR exam normal Your next step?
A. antibiotics
B. open and drain
C. laparotomy
Answer is: B, if there wound exploration as more accurate.

546. 60 years old man had an Open surgery and he noticed 20ml fluid come from
midline abdomen what’s your appropriate management
A. Dressing
B. Antibiotics
C. Ct abdomen
D. Wound exploration
Answer is: A

547. 25 y.o male came to ER with pelvic fracture, retrograde urethrogram done,
showed membranous urethra injury, what is the most appropriate NEXT step?
A. Suprapubic catheter
B. Surgery
Answer is: A

548. Pt did lap cholecystectomy before 2 weeks, now present with vague abdominal
pain, CT: large collection in subhepatic area, treatment?
A. operative drainage
B. CT-guided drainage
C. Laparoscopic drainage
Answer is: B

549. Pt with Afib he developed LL ischemia with intact femoral pulse and absent distal
pulse, impaired sensory and altered motor Next step?
A. CTA
B. Venous duplex
C. Embolectomy with no imaging
Answer is: C, if there arterial duplex is more accurate but Embolectomy is
indicated.

83
550. Case about calcified hydatid cyst, there was no observation in choices. Choices
were?
A. surgery
B. albendazole alone
C. another two unrelated
Answer is: Conservative treatment is more accurate, otherwise > A

551. 46 female done US for (i don’t remember) Found gallbladder polyp 6 cm what
next step?
A. Monitor with US after 6 months
B. Liver function test
C. CT
Answer is: if 6 cm > surgery, if 0.6 cm > A.

552. 20 years with femur fracture devitalized with dirt on wound best next step?
A. IV Abx
B. Surgical debridement
C. External fixation
D. Internal fixation
Answer is: A

553. Case typical signs and symptoms of Atelectasis (post-operative day 1) ask about
management?
A. Incentive spirometry
B. vasodilator
C. antibiotic
Answer is: A

554. 36 year come with abdominal distension, exaggerated bowel sound, history
sleeve gastrectomy 6 years ago?
A. adhesion
B. internal hernia
C. Other I exclude
Answer is: A

555. Post-operative of appendicitis 8 days ago, came in clinic with seroma?


A. Regular wound dressing
B. CT
Answer is: A

556. Pt did sleeve and comes after 4 wks severe abd Pain?
A. Exploratory
B. Imaging?
Answer is: stable > B (CT), unstable > A.

84
557. Biliary colic case presentation No leukocytosis Normal gall bladder wall, with
multiple stones, what u will do? Only mentioned that increase with fatty meal and
pain subsides after 2 hours and Radiates to shoulder?
A. Observation
B. Lap choly
C. ERCP
Answer is: medical treatment, if not there or Hight risk (bariatric pt or
hemolytic pt …) patient > B.

558. 6-year-old with femur fracture and 30-degree angulation?


A. Open reduction with IM
B. Open reduction with internal fixation (plate)
C. Closed reduction with Hip spica
D. Traction with …
Answer is: A

559. I have got 2 different scenarios with a question about which one is an indication
of thoracocentesis?
A. 1st Q: Lobulated pleural fluid
B. 2nd Q: Glucose < 60

560. After neck surgery, the patient felt numbness in skin lower half of the ear.
Physical examination showed loss of sensation of lower half of ear pinna and the
angle of the mandible. What is the affected nerve?
A. Third occipital nerve
B. Greater auricular nerve
C. Lesser occipital nerve
D. Greater occipital nerve
Answer is: B

561. Pt presented with typical scenario if Mallory weis, pt is stable what is the
management?
A. Endoscopy
B. Conservative
Answer is: B

562. Young man post hernial repair Came with small right testicle What is the cause?
A. Ligation of right iliac or sth artery
B. Mesh migration
C. Inguinal canal stricture
D. Thrombosis of pampiniform plexus
Answer is: D

85
563. Trauma pt in peripheral center, pthas flail chest, he is stable and u want to
transfer to trauma center 40 km away, what to do?
A. Intubate
B. chest tube
C. call and inform surgeon on call
Answer is: if the case is flair chest > A, but similar Q 2019 bank it's not flair
chest and there was aortic injury > C.

564. Farmer and has liver cyst (Echinococcus), 5x6 cm and calcified, Mx?
A. Surgery
B. Albendazole
C. PAIR
Answer is: conservative management if not there > A

565. Malignant Phelloyed breast tumor with no mets or LN, Mx?


A. WLEB
B. Chemo
C. Radiation
Answer is: Simple Mastectomy

566. Penetrating neck injury they mentioned zone 1 and pt was stable, next step?
A. CT of head and chest
B. exploration of wound
Answer is: A

567. Pt fall on stretched hand and cannot extend his DIP, what tendon injured?
A. flexor superficialis
B. flexor profundus
Answer is: B

568. Pt after Modified radical mastectomy has inner arm numbness, what nerve
injured?
A. long thoracic
B. thoracodorsal
C. intercostobrachial
Answer is: C

569. Asymptomatic varicose veins and the patient what to remove them, what is the
ttt?
A. Sclerotherapy
Answer is: A

86
570. 7 days post appendectomy presented with fecal discharge from the wound with
sign and symptoms of peritonitis vital sign stable except tachycardia and no fever?
What best next step?
A. IV antibiotic
B. wound exploration
C. CT abdomen
D. exploratory laparotomy
Answer is: A

571. Pt after ERCP developed abdomen chest neck subcutaneous emphysema what
most likely injury?
A. duodenal
B. esophagus
C. trachea
Answer is: A

572. Pt with malignant phylloid tumour what is next?


A. Contrast ct of chest
B. pet scan
Answer is: hormone receptor status, more accurate simple mastectomy.

573. case of intussusception came with colicky pain + doughnut sign on ultrasound +
bloody stool what is most important step to manage this case?
A. Urgent surgery
B. Nasogastric decompression
C. IV fluid resuscitation
D. Barium enema
Answer is: C

574. (Long case) patient has a lateral neck mass near the angle of the mandible,
Cervical lymph node. FNA: thyroid is normal and the mass has folicular cells. What
is the diagnosis?
A. Ectopic thyroid
B. Aberrant thyroid
C. Thyroid metastasis
Answer is: C

575. Patient had a neck dissection, now she can’t feel the lower lobe of her ear and
the mandible; physical examination decreased/loss of sensation in the lower
pinna and mandibular region of the same side. Which nerve most likely got
injured?
A. Third occipital
B. Greater auricle
C. Lesser occipital
D. Greater occipital
Answer is: B

87
576. Patient presented with abdominal pain, in exam there was a pulsatile mass.
What is most appropriate diagnostic test?
A. US abdomen
B. CTA
C. MRA
D. Conventional angiography
Answer is: A

577. PAD case, 100-meter claudication, DM heavy smoker long scenario not
important the question was how to improve his walking distance?
A. supervised exercise program
B. strict glycemic control
C. smoking cessation
Answer is: A

578. Pregnant lady with sx of cholecystitis asks u when it’s best to do the surgery?
A. First trimester
B. Second trimester
C. third trimester
Answer is: B, N.B: best.

579. Man with inguinal swelling in one side distinguishable from testicle with positive
cough impels, what is the diagnosis?
A. Spermatocele
B. Indirect inguinal hernia
C. Epididymal cyst
D. vaginal cele
Answer is: B

580. Patient with RUQ pain since 3 months, present with fever and jaundice (high
total bilirubin), next step in management?
A. ERCP
B. Abdominal cholecystectomy
C. Laparoscopic cholecystectomy
Answer is: according if cholangitis > A, if cholecystitis > C.

581. 55 y/o female did mammogram for screening, it was normal, she didn't have any
risk factor or family history of breast cancer, when is the time of next screening?
A. 1 year
B. 2 years
C. 3 years
D. 4 years
Answer is: B

88
582. Patient after RTA present with facial and basal skull fractures, what is the first
step in the management?
A. Support the airway
B. Support the neck
C. Neurological consultation
Answer is: A

583. PT after RTA present with pupil sluggish reaction to the light, eye movement
with pain, withdrawal and flexion of limbs with pain, and not understandable
words in response to pain, what is GCS?
A. 3
B. 6
C. 7
D. 9 (or 8 not sure)
Answer is: D

584. Pt. Comes post total gastrectomy with deficiency in Vit B?


A. B2
B. B6
C. B12
D. B1
Answer is: C

585. A child presented with one testicle and during exam you found the other in the
inguinal ring, what to do?
A. orchestom
Answer is: Orchiopexy

586. 8Y old boy. Mother complained of poor training to bath room Q: which muscles
is the target in therapy?
A. Perianal
B. pelvic floor
C. Rectus muscle
D. Detrusor
Answer is: B

587. Colorectal surgeon performing a low anterior resection for CRC and the pelvis
won’t stop bleeding, what you will do?
A. Observe
B. Infrarenal clamp
C. Supraceliac clamp
D. Pelvic packing
Answer is: start first with > D, if there was in Q packing and failed then > B.

89
588. Pt had RTA and give Sx of cardiac tamponade (rise JVP, distanced heart sound,
hypotension) and given report of CXR (there was cardiomegaly + broken
mediastinum) dx?
A. cardiac tamponade
B. heamomediastinum
Answer is: A

589. old female complaining of bone ache relived by drinking cold milk labs? high ca,
high PTH, low phosphate, low vitamin D, most likely Dx?
A. 1ry hyperparathyroidism
B. 2ry hyperparathyroidism
C. milk alkali syndrome
D. idiopathic hypercalcemia old
Answer is: Bad recall, but high PTH and Ca > A.

590. Female Pt came with leg swelling and darkening of the skin and intact pulses, no
history of travel or trauma what is the initial test?
A. MRV
B. dopplex venous
Answer is: B

591. Biopsy method for limb sarcoma?


A. incisional
B. Excisional
C. core biopsy
Answer is: C

592. Duodenal perforation best surgery?


A. Omentum patch
B. Jejongastrosomy
Answer is: A

593. Post appendectomy 4 days with peritonitis?


A. Laparotomy
B. Dressing
C. Antibiotics
Answer is: C

594. Ear infection with tympanic membrane perforation and pus come out of ear?
A. Acute otites media
B. Otites media with perfusion
Answer is: A

90
595. Patient had a dislocated shoulder the arm is fixed adducted and internally
rotated what type of dislocation is it?
Answer is: Posterior dislocation.
▪ Anterior > Abduction and external rotation.
▪ Posterior > Adduction and internal rotation.

596. Patient has sudden right eye pain, red with dilated pupil, cloudy cornea and
increased IOP (Intra ocular pressure); left eye by examination has cupping disc,
and normal IOP, which of the following is the likely diagnosis? NEW20
A. Right glaucoma, left glaucoma
B. Right uveitis and Left retinal degeneration disease
C. Right conjunctivitis and left reflex symptoms
Answer is: A
Right eye is glaucoma eye and left eye cupping and atrophy of optic nerve
mean there is glaucoma even IOP (Intraocular pressure) is normal, which
mean is chronic glaucoma in left eyes.

597. 70 yrs old man k/c of IHD, central abdominal pain vitally stable, amylase 600,
WBC normal, abdominal x ray: dilated small bowl with thickened wall: what’s the
dx?
A. Pancreatitis
B. Intestinal obstruction
C. Perforated ulcer
D. mesenteric vascular occlusion
Answer is: C

598. Patient come with severe 24h central abdominal pain and lab results show
elevated amylase. Radiology shows dilated thickened intestine. What is the most
likely diagnosis?
A. Intestinal perforation
B. Acute pancreatitis
C. Chronic pancreatitis
D. Gastritis
Answer is A

599. Obese Male 173cm 125 kg Htn dm failed lifestyle modification what next?
A. Medical treatment
B. Sleeve
Answer is: B

91
600. 60 years old male with history of DM and HTN and smoking, low compliance on
his medication, he presenting with pain in his right limb, absent of pulse, and cold
on examination What is most common causes will lead to death in this case?
A. MI
B. Stroke
Answer is A

601. pregnant at 16 wk complain of rt sided abdominal pain , all pregnancy causes


were excluded by obstetrician T:37.8 what’s the possible cause: in other recall
Pregnant patient has abdominal pain; fever; no labs?
A. gastritis
B. pancreatitis
C. cholecystitis
D. appendicitis
Answer is: D

602. Female pregnant...complain of right abdominal pain (did not specify upper or
lower) positive tenderness and rebound What’s the most likely diagnosis? (no labs
or images were done)?
A. Pancreatitis
B. Cholecystitis
C. Appendicitis
Answer is: C

603. Mean Arterial Pressure is 55, intracranial pressure 15. What's the cerebral
perfusion pressure?
A. 40
Answer is: A, CPP= MAP-ICP

604. Case of angiodysplasia: Large amount- pt is hypotensive?


A. Endoscopic laser
B. Conservative
C. Angioembolization
Answer is: C

605. 59 females with fatigue and paleness. Sometimes passed stool with streak
blood. Dx?
A. cecum cancer
B. sigmoid cancer
C. rectal cancer
D. bleeding from haemorrhoid 2nd degree
Answer is: C

606. this with case of 31 y o man with cauliflower anal mass with pain and itching?
A. HBV
B. condylomta acuminate
Answer is: B, N.B: if Old age, weight loss thinks of Anal cancer.

92
607. Patient after haemorrhoids operation develop Suprapubic pain with inability to
pass urine what is the cause?
A. Dehydration
B. urethral injury
C. Inadequate analgesic
Answer is: C

608. A case of swelling ant. Anal after laproscopic surgery, wound is clean?
A. US drainage
B. Abx 
C. Laparotomy 
D. Wound exploration 
Answer is: A

609. postpartum presented with breast pain on examination there's local erythema,
tenderness, WITH THINNING OF SKIN. How will you manage?
A. Observation
B. Incision and Drainage
C. Antibiotic
Answer is: B

610. 5 cm breast abscess in lactation mother, how to deal?


A. incision and drainage
B. u/s guided drainage
C. abx
Answer is: B, if there was thinning of skin > A.

611. Postpartum with a swelling redness breast lump the is mobile and lymph node is
palpable?
Answer is: Aspiration, if there was fluctuation or thinning of skin > I & D.

612. Female pt. with back pain With MRI dx mild spinal stenosis ttt?
A. Biofeedback
B. physical therapy
Answer is: B

613. Tenderness in ex decrease air entry on Rt side on us there's a collection 15*12 in


the gallbladder fossa?
A. Observation
B. Percutaneous aspiration
C. Physiotherapy
Answer is: B

93
614. Pt post PTC has fever, on PR exam ant. Boggy mx?
A. Percut Drainage IV Abx
B. IV paracetamol
Answer is: A

615. Pt developed bleeding after percutaneous transhepatic cholangiography also has


obstructive jaundice, what you will do next?
A. CT
B. US
C. Endoscopy
Answer is: C, if there was angiographic embolization is more accurate.

616. Old pt has compression fracture what is the first step?


A. Paracetamol
Answer is: A

617. 81 female Patient complaining of back pain after cough, x ray showed
compression fracture; what is the initial step in management?
A. Intranasal calcitonin
B. Paracetamol
C. Admit for surgery
D. Iv bisphosphonate
Answer is: B

618. Pt when you cover his Lt eye, the Rt eye go laterally?


A. Strabismus
B. Nystagmus
C. Amblyopia
Answer is: A

619. Child tripped on a toy and the right leg was trapped within the toy, and fell on
the leg child complaining of pain, what type of fracture do you expect?
A. Spiral facture of femur
B. spiral fracture of tibial
C. hairline fracture
Answer is: B

620. 6-year-old with a tibial fracture Closed with 30-degree angulation how to
manage?
A. traction with bed rest
B. closed reduction woth spica cast
C. open reduction with intramedullary nail
D. open reduction with plates
Answer is: C

621. supra condylar fracture in a child, distal pulse not palpable, what tx?
Answer is: close reduction and recheck the pulse again.

94
622. child with open fracture, management?
Answer is: Irrigation, debridement and fixation

623. Child with distal end of radius fracture (open 1 cm). What’s the management?
A. Debridement +antibiotic
B. Reduction and fixation
Answer is: A

624. 9 years old boy with a mass on the upper thigh that is expanding since 3 months
and not affecting his mobility. X-ray showed lytic lesion. What is the diagnosis?
A. Ewing sarcoma
B. Osteosarcoma
C. Chondrosarcoma
Answer is: B

625. Boy overweight have left hip pain after minor trauma “there was a pic”?
A. slipped capital femoral epiphysis
B. Femoral neck fracture
Answer is: A

626. Child with end hand fracture with small wand Mx?
A. debridement, operation
Answer is: A, N.B abx before debridement

627. child have supratrochlear fracture and absent radial pulse?


A. k wire
B. surgical exploration
C. close redaction
D. elevated
Answer is: C

628. Failed medical treatment of anal fissure?


Answer is: Lateral internal sphincterotomy

629. When to d erapy in hemorrhoids?


A. Internal
B. External
C. Prolapsed
Answer is: A

630. after sinusitis surgery affect sensation in lower eye and upper lip nerve?
Answer is: Infra-orbital

95
631. Pregnant pt come with painful breast mass, tender and redness, asking about
diagnosis?
A. Mastitis
B. Abscess
C. duct ectasia
D. something
Answer is: B

632. Patient young age k/c of lower limb AV malformation, which of the
following sx that will indicates intervention?
A. Pain
B. Claudication
Answer is A

633. Child’s hand picture with distal radial fracture with angulation and deformity
with lesion. whats next management?
A. close reduction with below elbow cast
B. close red with above elbow cast
C. surgical fixation
Answer is: C, If open wound >> irrigation and debridement first

634. 16y old with fever and RU pain and jaundice, No lab or radiology available?
A. Reassure and send home
B. Emergency consult a surgeon on phone
C. ask for lab results & tell him to come back tomorrow
D. admit to hospital & evaluate
Answer is: B

635. 17 yrs young athlete with pain? NEW20


A. exercise type
B. zinc def
C. Personal hygiene
Answer is: B

636. Lactating woman came with signs and symptoms of mastitis. What the next
appropriate management?
A. Flucloxacillin
B. I & D
Answer is A

96
637. A girl with vascular malformation of the left leg increasing with the years what
indicated surgery? In other recall, Child with vascular malformation of lower limb ,
when to interfere?
A. Pain
B. Claudication
C. Congestive heart failure
D. Cosmetic reason
Answer is: C

638. Pt with vascular malformation, which one is indicated for intervention?


A. Pain
B. Claudication
C. congenital heart disease
Answer is: Congestive heart failure

639. 25y old male with c/o of excessive wt gain since 5 ys wt 90 BMI 34 examination
normal?
A. start orlistat
B. investigate endocrine cause
C. 2ry cause of obesity
D. Diet and exercise
Answer is: D

640. Pt old hx recent multiple falls – imaging normal has dizziness?


A. post-concussion syndrome
Answer is: A

641. pt with spider nevi other things are normal about Dx?
A. US
B. clinical enough
C. MRV
D. CTA
Answer: B

642. during CS, upper abdominal bleed diagnosis?


Answer is: LIVER hemangioma.

643. old woman with bilateral leg heaviness and dragging PE sensation thick non
pitting edema dark skin what's inv?
A. duplex venous US
Answer is: A

97
644. A woman has spider veins in her anterior thigh. She wants to remove them
cosmetically. Before sclerotherapy what is the investigation to be done?
A. Venous duplex
B. No investigation needed
Answer is: A

645. Diabetic retinopathy with macular edema? NEW20


A. Panretinal photocoagulation
B. virotomy
Answer is: A

646. 55 years old female done mammogram now, when to do it again?


Answer is: after 2 years

647. Female presented with sudden acute onset of abdominal pain. US show mass in
the ovary. What’s the diagnosis?
A. Rupture of ovarian cyst
B. Ovarian torsion
Answer is: B
648. 2-year-old boy with pain over anterior tibial tubercle, diagnosis?
Answer is: Osgood Schlatter Disease

649. Peds 8 yrs old with RLQ pain and rebound tenderness what's confirmatory test?
A. US abdomen
B. MRI abdomen
C. CT abdomen
Answer is: A

650. Newborn circumcision, chordee and hooded foreskin + hypospadias, how to


manage?
A. open circumcision
B. plastibell circumcision
C. circumcision with
D. referral to pedia surgeon
Answer is: D

651. Pediatric patient presented with abdominal pain and jelly like stools, right
quadrant mass (case of intussusception) what’s the best diagnostic test?
A. Abdominal x-ray.
B. Abdominal CT.
C. Abdominal U/S.
D. Barium enema
Answer is: C

652. Child with X ray of distal radial and ulnar bone fractures?
Answer is: Cast below elbow

98
653. Male adult with perianal swelling and discharge, then ruptured, Dx?
A. Anal fissure
B. haemorrhoid
C. anal abscess
D. fistula
Answer is: D

654. Alcoholic patient known to have cirrhosis presents with massive upper GI bleed.
They admitted him for EGD. What is an important medication to give before
EGD?
Answer is: Octreotide

655. Anal fissure with skin tag, management?


A. Internal lateral sphincterotomy
B. Curettage of fissure with removal of the skin tag
Answer is: A

656. Pt 60 y/o long standing DM and HTN has pain in buttock and weakness of muscle
On CT angio show: - aortoiliac insufficiency. What is appropriate ttt?
A. vasodilator
B. Axillofemoral bypass
C. aortofemoral bypass
Answer is: C

657. 50-Young lady right lower abd pain intermittent (lasts 10-15min) with nausea
and vomiting on exam right lower quad fullness was felt,dx?
A. Ovarian torsion
B. Appendicitis
Answer is: A

658. Multigravida pt came with midline mass, apprears when leaning forward.
Negative cough impulse. Intact anterior abdominal fascia. What's the most likely
diagnosis?
A. Spigelian hernia.
B. Incisional hernia.
C. Divercation of recti.
D. Transversalis muscle weakness.
Answer is: C

659. Female 16 week pregnant, came with right sided abdominal pain associated with
nausea and vomiting. The doctor ruled out all pregnancy related abdominal pain.
What's the most likely diagnosis?
A. Gastritis
B. Pancreatitis
C. Cholecystitis
D. Appendicitis
Answer is: D

99
660. Stab wound in the chest. Came with weak thread pulses, raised JVP, EBAE.
What's the diagnosis?
A. Pneumothorax.
B. Cardiac tamponade.
C. Pulmonary contusion.
Answer is: B

661. Thigh wound, came with active bleeding. What's the next step?
A. Infusion of ringer lactate.
B. Tourniquet around the wound. (No applying pressure in choices)
Answer is: B

662. Pt diagnosed with thigh sarcoma. What the staging method?


A. Bone scan
B. CT of the chest
Answer is: B

663. Case of a child with intussusception. What's the most diagnostic?


A. Passage of jelly stool.
B. Palpation of a sausage shaped mass on abdomen.
Answer is: B

664. Elder pt diabetic and heavy smoker complaining of claudication after walking
100m. Distant pulses are intact but weak femoral pulse. What is the way to
increase his walking distance?
A. Glycemic control
B. Smoking cessation
C. Dual anti platelets
Answer is: Supervised walking program if not in choices B

665. ICU pt post operation complaining of high-grade fever with no other symptoms.
What's the management?
A. IV antibiotic
B. IV paracetamol
Answer is: A

666. Pt underwent mastectomy, complaining of numbness in medial aspect of the


arm. What's the nerve injured?
A. Long thoracic nerve
B. Intercostobrachial nerve
Answer is: B

100
667. patient did appendectomy 6 years ago cam with Symptoms of small bowl
obstruction X ray air fluids level Best diagnostic tool?
A. US
B. CT
C. laparoscopic exploration
Answer is: B

668. anterior neck injury neck tissue exposed, desaturated what to do?
A. high flow oxygen mask
B. cricothyroidotomy
C. intubation
Answer is: A

669. 6 years old Pediatric femur fracture and 30 angulation management?


A. Closed reduction and spica
B. open reduction and intramedullary nail
C. open reduction and plat
Answer is: B

670. Patient asymptomatic found to have gallbladder polyp 6mm Most appropriate
management?
A. No intervention required
B. Lap chole
C. Open chole
D. Follow up with US after 6 months
Answer is: D

671. Post op patient found to have erythema and mild swelling at wound site, no
discharge, no tenderness, no discharge (I can’t remember exactly if it was mild
tenderness or mild swelling but there’s nothing else) All vitals are normal even
temp was normal Dx?
A. Seroma
B. Abscess
C. Wound infection
D. Can’t remember
Answer is: A

672. Patient will do thyroid surgery with history of mitral valve prolapse and mitral
regurgitation?
A. Give prophylaxis abx for endocarditis
B. Give prophylaxis abx for wound infection
C. No indication for abx prophylaxis
Answer is: C

101
673. Elderly patient presented with bloody diarrhea and and exaggerated
bowel sounds X-Ray showed dilated descending colon (not sure) and thumb
print sign, dx?
A. UC
B. Crohn’s
C. Ischemic colitis
Answer is: C

674. Child came to ER with severe testicular pain for 5 hours, swelling, redness,
absent cremasteric reflex, most appropriate?
A. Surgical consult
B. US
C. Culture
Answer is: A

675. 5 y/o boy, his parents noticed that he has undescended testis, on examination
unilateral undescended testis, management? NEW20
A. Orchidopexy
B. Orchedectomy
C. Wait until puberty
D. Androgen hormone
Answer is: A

676. 20y/o female married, presents with severe RLQ pain with nausea and vomiting,
on exam there is rebound tenderness, Vitaly stable?
A. Uretrocele
B. Ovarian torsion
C. PID
D. Honeymoon cystitis
Answer is: B

677. Most common approach for mesh in ventral hernia?


A. Sublay
B. Inlay
C. Outlay
Answer is: A

678. 4 months old came for circumcision and you find mid-shaft hypospadias, what to
do?
A. Do it now
B. Not possible since they will use it for the repair,
C. Refere to surgery doctor
Answer: C

102
679. Elderly with progressive jaundice, abdominal discomfort and RUQ pain
Imaging showed dilated intrahepatic ducts and shrunken gallbladder, Dx? (can’t
remember the exact manifestations but sure about the imaging)
A. Klatskin tumor
B. Pancreatic cancer
C. CBD stones
Answer is: A

680. 12-year-old with nonspecific blunt trauma on his abdomen, later he presented
with generalized abdominal pain, imaging of the spleen showed 7 mm hematoma
and 4 cm tear (grade 3) (they wrote the grade in the Q) Most appropriate
management?
A. Splenectomy
B. Conservative
C. Spleen preserving surgery
Answer is: B

681. Adult after RTA he has aortic injury and spleen laceration, while you examine the
patient his BP drops (can’t remember the exact BP but he was unstable) Most
appropriate management?
A. Laparotomy
B. Conservative
C. Admit the patient in ICU
D. Transfer the patient to a centre
Answer is: A

682. 40 years old patient complain of anal pain after defecation, on examination
there is anal fissure with skin tag, management?
A. Lateral internal sphincterotomy
B. Lateral external sphincterotomy
C. Fistulectomy
Answer is: A.

683. Child (not sure) with severe RUQ pain for 2 hours which disappeared on its own
when he arrived to the hospital, what your diagnosis? NEW20
A. Mirrizi syndrome
B. Renal colic
C. Biliary colic
Answer is: C

103
684. Elderly with claudication pain after walking 100 meters when he is going to the
mosque, on examination: femoral pulse is intact and diminished distal pulses,
most appropriate management?
A. Surgical repair
B. Aortoiliac bypass
C. Stent placement
D. Heparinization
Answer is: D

685. Post RTA patient with humerus fracture which was managed by open reduction
and internal fixation, after surgery the patient can’t extend his forearm, wrist and
fingers Which nerve is injured?
A. Ulnar …
B. Radial …
C. Median …
D. Median ….
Answer is: B

686. 2 years old patient with umbilical hernia which protrude with cough and
disappear when lying down, management?
A. Lap with mesh
B. Open with mesh
C. Observe
Answer is: C
Can’t remember the choices exactly but there is no option wait until age of
5, only observe or surgical choices.

687. Child post trauma presented with open fracture of the leg, there is devitalized
tissue and dirt at the site of the fracture, first step in management?
A. Surgical debridement
B. Antibiotics
C. External fixation
Answer is: B
688. the first one 4x5 cm and they wrote the diagnosis (phyllodes), management?
A. Simple mastectomy
B. WLE
C. Chemo
D. Radio
Answer is: B

689. 35 or 45-year-old pregnant present with left side breast mass, firm, mobile, what
to do next?
A. Bilateral mammogram
B. Bilateral US
C. Core biopsy
D. Follow up after delivery
Answer is: B

104
690. Female presented with bloody nipple discharge and inverted nipple, what to do
next?
A. WLE
B. Core biopsy
C. Simple mastectomy
D. FNA
Answer is: A

691. Female post mastectomy complains of paraesthesia and numbness of the upper
arm and axillary Injury to which nerve?
A. Long thoracic nerve
B. Thoracodorsal nerve
C. Intercostobrachial nerve
D. Medial pectoral nerve
Answer is: C

692. Female complain of unilateral lower limb edema, bluish pigmentation, intact
femoral and distal pulses What is the next best step?
A. CT venography
B. US duplex
Answer is: B

693. Female complain of bilateral varicose veins (GSV) What is the most appropriate
investigation?
A. CT venography
B. US duplex (Forgot the rest)
Answer is: B

694. Patient with melena and sometimes blood per rectum, upper and lower
endoscopy were normal, what to do next?
A. Endoscopy capsule
B. Isotope scan
Answer is: A, Can’t remember the rest but no tec-99 or tagged RBCs)

695. Pt came after accident Ex finding neck injury with tissue decrease oxygen sat
Next step?
A. Oxygen mask
B. Mechanical ventilation
C. Cricothyrotomy
Answer is: A
It’s too invasive and his neck is injured it will be more difficult Since he
is desating provide him with oxygen first

105
696. Case of spleen injury Grade 3 and pt stable ask about Mx?
A. Conservative Mx
B. Open surgery
Answer is: preserving surgery
▪ Grade 1, 2 > non operative management
▪ Grade 3,4,5 > operative management
▪ Stable > preserving surgery (splenorrhaphy)
▪ Unstable > splenectomy

697. Colorectal surgeon performing a low anterior resection for CRC and the pelvis
won’t stop bleeding, so he consults a vascular surgeon and he did pack and it
doesn’t help what you will do?
A. Observe
B. Infrarenal clamp
C. Supraceliac clamp
D. Do the surgery and remove packing
Answer is: B

698. patient did appendectomy 6 years ago cam with Symptoms of small bowl
obstruction X ray air fluids level Best diagnostic tool?
A. US
B. CT
C. laparoscopic exploration
Answer is: B

699. 12 years old male with testicular pain (horizontally lying, mildly elevated, no
swelling or erythema of scrotum) what to do next?
A. surgical consultation
B. rehydration
C. US
Answer is: A

700. When say this pain from organ (organic pain)?


A. Before sleep time.
B. Before awakening.
C. Two choices I forgot it
Answer is: B

701. 40 years Pt with hx of bloody discharge from breast? P/E normal Initial invest?
A. US
B. Mammogram
C. MRI
Answer is: B

106
702. A 50-year-old lady presents with bloody nipple discharge, and it was found to be
invasive intraductal papilloma. The most appropriate management is?
A. Wide local excision
Answer is: A

703. Female with 2×2cm benign phylloid tumor what is the management?
A. observation and follow up
B. WLE
C. simple mastectomy
Answer is: B

704. Infected Pancreatic pseudocyst?


A. Percutaneous drainage
Answer is: A

705. Patient with jaundice, abd pain and on US u see stones and dilated CBD vitals
show fever only asking about diagnosis?
A. ascending cholangitis
B. choledocholithiasis
C. cholecystitis
Answer is: A

706. Post percutaneous transhepatic cholangiography developed upper gi bleeding,


invx?
A. Endoscopy
B. CT angio
C. US
Answer: B

707. Old man suddenly had vomited two episodes of bloody vomit, Physical Exam
shows no Tenderness over his abdomen or sternal area, he is stable, what is your
diagnosis?
A. Mallory weiss tear
Answer is: A

708. Colorectal surgeon performing a low anterior resection for CRC and the pelvis
won’t stop bleeding, so he consults a vascular surgeon and he does?
A. Heavy packing of pelvis
B. Arteriography intra operatively
C. Infraceliac clamp
Answer is: B

709. th day post OP (appendicitis) presented with diffuse abd pain, distension,
vomiting and sluggish bowel sounds Abd xray: Multiple air/fluid level?
A. paralytic ileus
Answer is: A

107
710. 55 years old male presented with sudden abdominal pain radiated to the back
the patient has history of cardiomyopathy, the patient mentioned that he passed
small amount of loose stool, during the examination there is exaggerated bowel
sound Labs: Amylase = mildly elevated above normal What is the Diagnosis?
A. Acute pancreatitis
B. Mesenteric vascular ischemia
C. Ruptured aneurysm
Answer is: B

711. Picture of green stick and ask for treatment?


A. close reduction with cast
Answer is: A

712. Pt 56 c/ o sudden RLQ pain after lifting heavy object; and mass in RLQ; Cough
negative; abd muscle tense even with clenching. He is on anticoagulation due to A
fib What the most appropriate mgx? (Rectus sheath hematoma)
A. angioembolization and …
B. rest and analgesic
Answer is: B

713. 29 Years old male did sleeve gastrectomy 6 days ago, came to the ER
complaining of persistent vomiting since operation, on exam nothing significant,
ABG normal (there were numbers), Us normal. Best management?
A. Reassure
Answer is: A

714. Old male with htn and dm presented with claudication examination reveals right
femoral intact pulse and diminished popliteal and distal pulse and left diminished
pulse most important intervention?
A. CT angio
B. conventional Angio
C. MRA
D. venous US
Answer is: A

715. Bilateral great saphenous vein mange?


A. sclerotherapy
B. something endoscopic
C. ablation
Answer is: C

716. Neck trauma in area retro auricular above the angle of the mandible (area 1 was
not mentioned), patient stable and oriented. Most appropriate step?
A. CT angio.
B. Neck exploration
Answer is: A

108
717. After herniotomy surgery for 5yrs boy came with fever and pus discharge and
part of mesh seen?
A. give iv antibiotics
B. draining of pus
C. draining of pus and remove mesh
D. observation
Answer is: C

718. Bilateral inguinal hernia, how to manage?


A. lap repair with mesh
B. open repair with mesh
C. lap repair without mesh
Answer is: A

719. urn case black soot over nostrils and mouth 40% carboxyhemoglobin. (Carbon
monoxide toxicity)?
A. Hyperbaric oxygen.
B. Intubation and ventilation with 100% o2.
C. Carbonic anhydrase inhibitors
Answer is: B

720. Patient had a hyperextension trauma, complains of distal phalanx pain and
tenderness in the volar aspect, he also feels tenderness in the palm, what's the
dx?
A. Rupture of flexor profundus
B. Rupture of flexor superficialis
C. intra-articular fracture of proximal phalanx
D. extra-articular fracture of distal phalanx
Answer is: A

721. t presented with many bouts of hematemesis after prolonged vomiting, Dx?
A. Mallory weiss
B. Esophageal varices
Answer is: A

722. pancreatitis case managed conservatively at the day 3 the pt improving clinically
and laboratory. Everything normal except Amylase 250. US showed>> Gallstone at
the gall bladder with dilated extra hepatic ducts (nothing was mentioned about
CBD). What is the appropriate management?
A. lap chole before discharge
B. elective lap chole
C. ERCP
Answer is: A

109
723. Elderly male pt , resident of nursing home with hearing aid presented with
cerumen impaction. Examination revealed bilateral Rene test + and Weber
lateralized to the right, what is the most likely dx?
A. conductive hearing loss more in the right ear
B. presbycusis is more the right ear
C. chronic otitis media in the left
D. Acoustic neuroma in the right
Answer is: A

724. patient present with presentation of cholangitis epigastric pain I think (clinically
not very typical but from the labs it looks like cholangitis). NO vitals provided. His
labs show High ALP, direct bilirubin. What’s the most important initial
management?
A. IVF
B. ABx
C. abdominal US
D. ERCP
Answer is: A

725. Old man came to you with Inguinal reducible hernia, and he was advised to do
hernia repair, and PE exam was normal he is asymptomatic what will you do?
A. no surgical treatment
B. open repair with mesh
C. Laparoscopic repair (no mesh mentioned)
Answer is: A, NOTE: There are no symptoms + Old + Inguinal Reducible.

726. 60 years old man had an Open surgery and he noticed 20ml fluid come from
midline abdomen what’s your appropriate management?
A. Dressing
B. Antibiotics
C. Ct abdomen
D. Wound exploration
Answer is: A

727. Man come to the ER for poly Truman and he was intubated after resuscitation
admitted to the ICU what is the best approach to know cervical spine injury?
A. MRI neck
B. Ap and lateral x-ray neck
C. Clinical judgment
D. CT nec
Answer is: D

110
728. 8 years old boy diagnosed DM type 1 one month ago when to see the
ophthalmologist?
A. 1 year
B. 2 years
C. 3 years
D. 5 years
Answer is: D

729. known case of Crohn came with multiple strictures in biliary tract?
A. primary sclerosing cholangitis
Answer is: A

730. after open inguinal hernia repair came with decrease size of one testis?
A. Ischemic orchitis secondary to pampiniform plexus thrombosis
Answer is: A

731. Which structure pass through inguinal ring?


A. Ilioinguinal
Answer is: A

732. Patient bleeding from angiodysplasia left side, Mx?


A. Left hemicolectomy
B. Angio-embolization
C. Endoscopic laser ablation
Answer is: Stable > C, Unstable > B.

733. 24 years old with hx of appendectomy 5 years ago present with abdominal pain,
distention, vomiting for 3 days, CT scan show obstruction signs and peritonitis
what is contraindicated in this pt?
A. Propofol
B. Ketamine
C. Sevoflurane
D. Nitrous oxide
Answer is: D

734. after accident found to have air under the diaphragm & spleen laceration &
thoracic aortic perforation (and hypotension) what first thing to do?
A. Thoracotomy
B. Exploratory Laparotomy
Answer is: B

735. child with above knee mass?


A. chondrosarcoma
B. Osteosarcoma
Answer is: B, N.B: Distal femur or proximal tibia think osteosarcoma.

111
736. Obese diabetic hypertensive female patient tried lifestyle but it was not
effective. BMI is 28. What is the best drug with tolerable side effects?
A. Orlistat
B. Lorcaserin
C. Phentermine-topiramate
D. Forgot it
Answer is: A

737. In a patient post mastectomy, they do for for her reconstruction from rectus
muscle, what is the vessels to be injured or effected? NEW20
A. Inferior Epigastric artery
B. Superior Epigastric artery
C. Intercostal artery
D. Internal thoracic artery
Answer is: B

738. Patient “forgot age” known history of diabetes mellitus, complaining of ptosis,
inability to adduct, elevate and depress right eye. Upon examination pupils are
reactive bilaterally what is the most common cause?
A. posterior communicating artery aneurysm
B. cavernous sinus thrombosis
C. 3rd nerve neuropathy
Answer is: C due to DM.

739. 26 y/o female with multiple masses felt in the left breast, gets tender around
period. U/S revealed multiple small cystic lesions and two solid masses?
A. Fibroadenomas
B. Fibrocystic
C. Breast cancer
Answer is: B

740. Patient know case of crohn’s disease came with abdominal pain there was intra
large intra-abdominal collection and iliujujenl fistula how to manage?
A. Percutaneous drainage
B. Open drainage with fistula repair
C. Open drainage
Answer is: A

741. Case of hydatid cyst with daughter cysts, what is the management?
A. deroofing surgery
B. Percutaneous drainage
C. IV Alparanazole alone
Answer is: A

112
742. Fracture of femur and angulation in 6-year-old what is the management?
A. Open reduction & intramedullary nail
B. Open reduction & plate
C. Close reduction and spica
Answer is: A

743. 50 y women no family Hx of breast ca , mammogram normal When to do next


mammogram?
Answer is: 2 years

744. The most risk factor of breast ca?


A. Age
B. Late menepose c
C. Early menarch
D. Get pregn
Answer is: A

745. Boy came with deep stabbed wound in the anterior right thigh 10 cm in depth.
What is your next step?
A. tourniquet
B. call vascular surgeon
C. direct pressure apply on the wound
D. directly clamp the femoral artery
Answer is: C

746. 12 years old male with testicular pain (horizontally lying, mildly elevated, no
swelling or erythema of scrotum) what to do next?
A. surgical exploration
B. rehydration
Answer is: A

747. Hernia what will you do after stabilization?


A. NGT
Answer is: A, according to full scenario and rest of answer.

748. Median nerve injures?


A. Ape hand
B. Claw hand
C. water tip hand
Answer is: A

749. Pt. presented with many bouts of hematemesis after prolonged vomiting, Dx was
Mallory weis what is your management?
Answer is: Conservative management

113
750. 29 years old presented with abd distention and pain, he was found to have
intssupation, wat is the Mx?
A. Surgical resection
B. Surgical decompression
C. Colonoscopy q
D. Enema
Answer is: A

751. DM and HTN known case patient, presents with severe epigastric pain radiating
to back for 6 hours, CXR shows left air under diaphragm, most likely diagnosis?
A. Perforated peptic ulcer
Answer is: A

752. Double vision, when he look straight the lt eye deviated to nose side, Lt. eye
cannot look laterally?
A. Rt. 6 CN palsy.
B. Rt. 3 CN palsy.
C. Lt. 6 CN palsy
D. Lt. 3 CN palsy
Answer is: C

753. Patient had thenar muscle atrophy however his sensation was intact over that
area, what nerve injury is most likely?
A. Radial
B. Median
C. Axillary
D. Ulnar
Answer is: B

754. pt. elderly with central abdominal pain, CT show small bowel cancer 3*4 with
calcification, what is the type cancer? NEW20
A. Schwannoma
B. Carcinoid tumor
C. Lymphoma
Answer is: B

755. pt. did lap cholecystectomy, come two week with chest pain, what most
appropriate next?
A. US
B. Catheterization
C. CT angio
Answer is: ECG, if not there > C

114
756. Patient asymptomatic found to have gallbladder polyp 6mm Most appropriate
management?
A. No intervention required
B. Lap chole
C. Open chole
D. Follow up with US after 6 months
Answer is: D

757. Post op patient found to have erythema and mild swelling at wound site, no
discharge, no tenderness, no discharge (I can’t remember exactly if it was mild
tenderness or mild swelling but there’s nothing else) All vitals are normal even
temp was normal? Dx?
A. Seroma
B. Abscess
C. Wound infection
D. Can’t remember
Answer is: A

758. Elderly patient presented with bloody diarrhea and and exaggerated bowel
sounds X-Ray showed dilated descending colon (not sure) and thumb print sign,
dx?
A. UC
B. Crohn’s
C. Ischemic colitis
Answer is: C

759. Child came to ER with severe testicular pain for 5 hours, swelling, redness,
absent cremasteric reflex, most appropriate?
A. Surgical consult
B. US
C. Culture
Answer is: A

760. 5 y/o boy, his parents noticed that he has undescended testis, on examination
unilateral undescended testis, management?
A. Orchidopexy
B. Orchedectomy
C. Wait until puberty
D. Androgen hormone
Answer is: A

761. Most common approach for mesh in ventral hernia?


A. Sublay
B. Inlay
C. Outlay
Answer is: A

115
762. 4 months old came for circumcision and you find mid-shaft hypospadias, what to
do?
A. Do it now
B. Not possible since they will use it for the repair
Answer is: B

763. Elderly with progressive jaundice, abdominal discomfort and RUQ pain Imaging
showed dilated intrahepatic ducts and shrunken gallbladder, Dx? (can’t remember
the exact manifestations but sure about the imaging)
A. Klatskin tumor
B. Pancreatic cancer
C. CBD stones
Answer is: A

764. 12-year-old with nonspecific blunt trauma on his abdomen, later he presented
with generalized abdominal pain, imaging of the spleen showed 7 mm hematoma
and 4 cm tear (grade 3) (they wrote the grade in the Q) Most appropriate
management?
A. Splenectomy
B. Conservative
C. Spleen preserving surgery
Answer is: C

765. Adult after RTA he has aortic injury and spleen laceration, while you examine the
patient his BP drops (can’t remember the exact BP but he was unstable) Most
appropriate management?
A. Laparotomy
B. Conservative
C. Admit the patient in ICU
D. Transfer the patient to a center
Answer is: A

766. 40 years old patient complain of anal pain after defecation, on examination
there is anal fissure with skin tag, management?
A. Lateral internal sphincterotomy
B. Lateral external sphincterotomy
C. Fistulectomy
Answer is: A

767. Child (not sure) with severe RUQ pain for 2 hours which disappeared on its own
when he arrived to the hospital, what your diagnosis?
A. Mirrizi syndrome
B. Renal colic
C. Biliary colic
Answer is: C

116
768. Elderly with claudication pain after walking 100 meters when he is going to the
mosque, on examination: femoral pulse is intact and diminished distal pulses,
most appropriate management?
A. Surgical repair
B. Aortoiliac bypass
C. Stent placement
Answer is: correct answer is D, heparization (in other recall).

769. Post RTA patient with humerus fracture which was managed by open reduction
and internal fixation, after surgery the patient can’t extend his forearm, wrist and
fingers Which nerve is injured? Sorry I can’t remember the rest of the sentences
but they mentioned 2 branches of median nerve
A. Ulnar …
B. Radial …
C. Median …
D. Median ….
Answer is: B

770. 2 years old patient with umbilical hernia which protrude with cough and
disappear when lying down, management?
A. Lap with mesh
B. Open with mesh
C. Observe
Answer is: C

771. Child post trauma presented with open fracture of the leg, there is devitalized
tissue and dirt at the site of the fracture, first step in management?
A. Surgical debridement
B. Antibiotics
C. External fixation
Answer is: B

772. 35 or 45-year-old pregnant present with left side breast mass, firm, mobile, what
to do next?
A. Bilateral mammogram
B. Bilateral US
C. Core biopsy
D. Follow up after delivery
Answer is: B

773. Female presented with bloody nipple discharge and inverted nipple, what to do
next?
A. WLE
B. Core biopsy
C. Simple mastectomy
D. FNA
Answer is: B, should be imaging.

117
774. Female post mastectomy complains of paraesthesia and numbness of the upper
arm and axilla Injury to which nerve?
A. Long thoracic nerve
B. Thoracodorsal nerve
C. Intercostobrachial nerve
D. Medial pectoral nerve
Answer is: C

775. Female complain of bilateral varicose veins (GSV) What is the most appropriate
investigation?
A. CT venography
B. US duplex
Answer is: B

776. Female post cholecystectomy complains of chills and rigors, all labs normal (BP,
Temp, HR) What is the cause?
A. Septic shock
B. Sepsis
C. Bacteraemia
D. SIRS
Answer is: D

777. Patient with melena and sometimes blood per rectum, upper and lower
endoscopy were normal, what to do next?
A. Endoscopy capsule
B. Isotope scan
C. Can’t remember the rest but no tec-99 or tagged RBCs)
Answer is: A

778. 28- female pt came with complex regional pain syndrome, she has hx of carpal
tunnel syndrome surgery, what is the Mx?
A. physiotherapy
B. Opioid
Answer is: A

779. Old pt presented to you with sign and symptoms of portal htn with esophageal
varices and did endoscopic ligation Later they discovered enlarged spleen with
splenic vein thrombosis what is your management?
A. splenectomy
B. splenio something bypass
C. total Portosystemic shunt
D. sengastaken-blakemore tube
Answer is: A

780. 6-19y with breast mass that is moblie and small and well localize, what is Dx?
Answer is: Fibroadenoma

118
781. 7-47y with breast mass that was small and enlarging whit time, thin skin b.c of
enlargement but no inflammation, what is the Dx?
Answer is: Phyllodes

782. 8-other similar q and they gave They result show phyllodes what to do?
A. WLE
B. simple mastectomy
Answer is: Depends on the size
783. 13-35y old pt worried his mother had. Breast Ca and his sister had ovarin Ca.
How to screen her?
A. Mamo
B. Us
C. BARCA
Answer is: C

784. 3-colorectal Ca screen age?


Answer is: 50 y

785. Most common cancer screen in asymptotic?


Answer is: Colorectal Ca

786. old pt had open appendectomy after complicated rapture appendix, after 6 w he
came with sing of intestine obstruction. And sign of peritonitis (they said sing of
peritonitis Only like that). And provide some lab, what is appropriate step?
A. exploratory laparotomy
B. CT
Answer is: A

787. Patient present with cholangitis epigastric pain I think (clinically not very typical
but from the labs it looks like cholangitis). NO vitals provided. His labs show. High
ALP, direct bilirubin What’s the most important initial management?
A. IVF
B. Abx
C. abdominal US
D. ERCP
Answer is: A

788. How to find appendix?


Answer is: follow taenia coli

119
789. Splenic vein thrombosis with gastric varices: splenectomy Other recall: Patient
after pancreatitis episode develops upper GI bleeding picture, scope was done,
gastric fundus bleeding was found, sclerotherapy done Duplex ultrasound
showed: splenic vein thrombosis with patent portal vein what is your
management?
A. Splenectomy
B. Portal-systemic shunt
Answer is: A

790. Old man came to you with Inguinal reducible hernia, and he was advised to do
hernia repair, and PE exam was normal he is asymptomatic what will you do?
A. no surgical treatment
B. open repair with mesh
C. Laparoscopic repair (no mesh mentioned)
Answer is: A, NOTE: There are no symptoms + Old + Inguinal Reducible

791. 60 years old man had an Open surgery and he noticed 20ml fluid come from
midline abdomen what’s your appropriate management?
A. Dressing
B. Antibiotics C: Ct abdomen
C. Wound exploration
Answer is: A (I would choose A, since no signs of infection and it needs
regular dressing)

792. Man come to the ER for poly Truman and he was intubated after resuscitation
admitted to the ICU what is the best approach to know cervical spine injury?
A. MRI neck
B. Ap and lateral x-ray neck
C. Clinical judgment
D. Ct neck
Answer is: D

793. after open inguinal hernia repair came with decrease size of one testis?
Answer is: Ischemic orchitis secondary to pampiniform plexus thrombosis!!!

794. Which structure pass through inguinal ring?


Answer is: Ilioinguinal

795. Patient bleeding from angiodysplasia left side, Mx?


A. Left hemicolectomy
B. Angio-embolization
C. Endoscopic laser ablation
Answer is: (Stable: c |Unstable: B)

120
796. pancreatitis case managed conservatively at the day 3 the pt improving clinically
and laboratory. Everything normal except Amylase 250. US showed>> Gallstone at
the gall bladder with dilated extra hepatic ducts (nothing was mentioned about
CBD). What is the appropriate management?
A. lap chole before discharge
B. elective lap chole
C. ERCP
Answer is: (If there’s no evidence of CBD stone (jaundice, dilated CBD,
elevated ALP) > no need for ERCP)

797. Clear scenario of mastitis with no fluctuations what to give?


Answer is: ABx Dicloxacillin

798. Elderly male pt, resident of nursing home with hearing aid presented with
cerumen impaction. Examination revealed bilateral Rene test + and Weber
lateralized to the right, what is the most likely dx?
A. conductive hearing loss more in the right ear
B. presbycusis is more the right ear
C. chronic otitis media in the left
D. Acoustic neuroma in the right
Answer is: Supposed to be presbycusis is more the left ear. But by
exclusion, go with C.

799. child with above knee mass?


Answer is: Osteosarcoma

800. In a patient post mastectomy, they do for for her reconstruction from rectus
muscle, what is the vessels to be injured or effected?
A. Inferior Epigastric artery
B. Superior Epigastric artery
C. Intercostal artery
D. Internal thoracic artery
Answer is: B

801. 24 years old with hx of appendectomy 5 years ago present with abdominal pain,
distention, vomiting for 3 days, CT scan show obstruction signs and peritonitis.
What is contraindicated in this pt?
A. Propofol
B. Ketamine
C. Sevoflurane
D. Nitrous oxide
Answer is: D

121
802. After accident found to have air under the diaphragm & spleen laceration &
thoracic aortic perforation and hypotension what first thing to do?
A. Thoracotomy
B. Exploratory Laparotomy
Answer is: B

803. Which of the following should be screened in general population?


A. Colorectal cancer
B. Ovarian cancer
Answer is: A

804. Anterior thigh stab & sever bleeding, “labs show little low hg in 80 & NO vital
signs” what’s the initial steps? NO direct pressure in the options
A. ringer lactate IV Fluid
B. Tourniquet
Answer is: B by exclusion

805. Patient with a stab wound in the right thigh and massively bleeding brought to
the emergency department unconscious and O/E there is active bleeding. What is
the most important next step to do?
A. Ringer lactate iv fluid
B. Blood transfusion
C. Tourniquet on the thigh
D. Orotracheal intubation
Answer is: D

806. Pt presented with stabbed wound after wound exploration you found anterior
abdominal fascia penetration, (his vitals were stable) what's your next step?
A. CT abdomen
B. MRI abdomen
C. Exploratory laparotomy
D. Diagnostic laparoscopy
Answer is: D

807. Female get high energy accident (high velocity), with seat belt sign, on X ray have
chance fracture, what will associated with this fracture?
A. Duodenal perforation
B. Gastric perforation
C. Jejunum perforation
D. Vena cava perforation
Answer is: A

122
808. 65 YO female, with change with bowl habit only (no weight loss or any cancer
signs)
A. Anal cancer
B. Anal warts cauliflower
Answer is: incomplete Q, change of bowel habit is symptom colon cancer,
if there was rectal cancer is more accurate.

809. 3 YO child fall from the bed, no LOC, after few hours had vomiting & headache &
cry, what you will do?
A. CT
B. MRI
C. Observation
Answer is: C

810. A 30 years old male present to E.R. complaining of visual deterioration for 3 days
of Rt. Eye followed by light perception, the least cause is? NEW20
A. Retinal detachment.
B. Central retinal arterial embolism.
C. Vitreous haemorrhage.
D. Retro-orbital neuritis.
E. Retinitis pigmentation.
Answer is: D

811. Child with above knee mass?


A. chondrosarcoma
B. Osteosarcoma
Answer is: B

812. In a patient post mastectomy, they do for her reconstruction from rectus
muscle, what is the vessels to be injured or effected?
A. Inferior Epigastric artery
B. Superior Epigastric artery
C. Intercostal artery
D. Internal thoracic artery
Answer is: B

813. Screening of low risk male colon cancer at which age?


Answer is: 50

814. Patient bleeding from angiodysplasia left side, Mx?


A. Left hemicolectomy
B. Angio-embolization
C. Endoscopic laser ablation
Answer is: stable > C, unstable > B.

123
815. Above renal verge 2cm cauliflower appearance on protocope - friable to tough?
A. Renal cancer
B. Rectal cancer
C. Something polyps
D. condyloma acuminata
Answer is: B

816. Breast nodule. With tethring skin and axillary LN involvemtn? BIRAD4 what to
do?
A. Needle core biposy
B. Excsional piopsy
C. Needle aspiration
D. Mastectomy
Answer is: A

817. MI and peri collection no aberrant stone in biliary tenderness in palpation fever.
What to do?
A. Emergency cholcyatomy
B. Percutation drainage
C. ERCP
Answer is: B

818. A 15 yo came with groin pain and lower flank pain for (4h) or (8h) P/E: the testes
is slightly ascended and tender in touch and no fever. And Redness. What the
appropriate next step?
A. Doppler US
B. Expiratory surgery
C. Abx
Answer is: if was horizontal > B, if not > A.

819. Pt bleeding (angiodysplasia cause) pt was hypotsive in80/50s. What to do?


A. Angioembolization
B. Laser ablation
C. hemicolectomy
Answer is: A

820. Post appendectomy patient presented with seroma collection, what the
appropriate management?
A. Regular dressing
B. Wound Exploration
C. US guided drainage
Answer is: A

124
821. Obese patient diagnosed with hiatel hernia, which of the following is best
operation for wight reduction?
A. Roux en Y
B. Sleeve gastrectomy
Answer is: A

822. case of gun shot, Raised JVP and no heart sounds, best appropriate
management?
A. Pericardiocentesis
B. Thoracotomy
Answer is: A

823. Patient had a long flight then admitted complaining of SOB, on examination
patient is tall, investigation showed pneumothorax, what is the appropriate
management?
A. Chest tube
B. Conservative management
Answer is: A

824. 60 years old man had an Open surgery and he noticed 20ml fluid come from
midline abdomen what’s your appropriate management?
A. Dressing
B. Antibiotics
C. CT abdomen
D. Wound exploration
Answer is: A

825. 24 years old with hx of appendectomy 5 years ago present with abdominal pain,
distention, vomiting for 3 days, CT scan show obstruction signs and peritonitis
what is contraindicated in this pt?
A. Propofol
B. Ketamine
C. Sevoflurane
D. Nitrous oxide
Answer is: D

826. pt with diplopia, ptosis, deviation of the right eye to the right?
A. oculomotor nerve
Answer is: A

125
827. A 62-year-old man with no significant medical history presented to Emergency
room with 1- day history of 4. haematochezia, He denies history of abdominal
pain, constipation or weight loos Blood pressure 90/50 mmHg Heart rate 112
/min, which of the following is the most likely diagnosis?
A. Cecal cancer
B. diverticulosis
C. Internal piles
D. Ischaemic colitis
Answer is: B

828. Scar post-surgery increases in size Extend beyond border?


A. keloid
Answer is: A

829. 60 years old man had an Open surgery and he noticed 20ml fluid come from
midline abdomen what’s your appropriate management?
A. Dressing
B. Antibiotics
C. CT abdomen
D. Wound exploration
Answer is: A

830. Case of nephrotic syndrome complaining of abdominal pain, diarrhea and


vomiting, with rebound tenderness, what most likely diagnosis?
A. Gastritis
B. Peritonitis
Answer is: B

831. 70 yrs old man k/c of IHD, central abdominal pain vitally stable, amylase 600,
WBC normal, abdominal x ray: dilated small bowl with thickened wall: what’s the
dx?
A. Pancreatitis
B. Intestinal obstruction
C. Perforate d ulcer
D. mesenteric vascular occlusion
Answer is: D

832. Female with benign phyllodes tumour asking about management?


A. Wide local excision
B. Wide local excision with radio
C. Mastectomy
D. Reassurance
Answer is: A

126
833. DM and HTN known case patient, presents with severe epigastric pain radiating
to back for 6 hours, takes NSAID for long time, CXR shows left air under
diaphragm, most likely diagnosis?
A. Perforated peptic ulcer
Answer is: A

834. pregnant 10 weeks complaining of biliary colic in the past 5 weeks Mx?
A. laparoscopic cholecystectomy now
B. laparoscopic cholecystectomy after delivery
C. laparoscopic cholecystectomy in second trimester
D. laparoscopic cholecystectomy in third trimester
Answer is: A, if ask about best time > C.

835. Child with bilateral inguinal hernia what is the treatment?


A. Herniotomy
B. laparoscopic mesh repair
Answer is: A

836. Pediatric patient presented with abdominal pain and jelly like stools, right
quadrant mass (case of intussusception) what’s the best treatment?
A. radiological enema
Answer is: A

837. Patient had thenar muscle atrophy however his sensation was intact over that
area, what nerve injury is most likely?
A. Radial
B. Median
C. Axillary
D. Ulnar
Answer is: B

838. 30 years morbidly obese male, how to decide best reduction surgery he will
have?
A. Barium enema
B. CT abdomen
C. ultrasound abdomen
D. GI endoscopy
Answer is: D

127
128

You might also like